RD respiratory formatted Flashcards

1
Q
  1. Dyspnea on exertion, weight loss, clubbing. CT chest shows nodular septal thickening, asymmetric. What is the most likely diagnosis? (also recalled as beading along bronchovascular bundles, lymphadenopathy & pleural effusions)
    a. Sarcoid
    b. Lymphangitic carcinomatosis
    c. Extrinsic allergic alveolitis
    d. Idiopathic pulmonary fibrosis
A

b. Lymphangitic carcinomatosis = classic findings

How well did you know this?
1
Not at all
2
3
4
5
Perfectly
2
Q
  1. Dyspnea on exertion, inspiratory crackles. CT chest shows bilateral ground glass and centrilobular nodules. What is the most likely diagnosis
    a. Pulmonary oedema
    b. Hypersensitivity pneumonitis
    c. Pneumocystis pneumonia
A

b. Hypersensitivity pneumonitis T upper>lower zone, centrilobular nodules, patchy GGO, headcheese sign

  1. Dyspnea on exertion, inspiratory crackles. CT chest shows bilateral ground glass and centrilobular nodules. What is the most likely diagnosis
    a. Pulmonary oedema T possible, esp. if perihilar & LZ predominance. Usually no abnormal physical findings when purely interstitial oedema.

b. Hypersensitivity pneumonitis T upper>lower zone, centrilobular nodules, patchy GGO, headcheese sign
c. Pneumocystis pneumonia F centrilobular nodules less common

How well did you know this?
1
Not at all
2
3
4
5
Perfectly
3
Q
  1. Which of the following is not a recognised feature of primary pulmonary amyloidosis:

a. Nodules in tracheobronchial tree
b. Bronchopleural fistula
c. Peripheral parenchymal nodules
d. Adenopathy

A

b. Bronchopleural fistula
Pulmonary amyloidosis
• Nodules in tracheobronchial tree
• Parenchymal nodules – peripheral/subpleural
• Adenopathy[Dahnert]
[SK – added a, c & d – only one option recalled]

How well did you know this?
1
Not at all
2
3
4
5
Perfectly
4
Q
  1. Lung disease, which does not cavitate.
    a. Small cell cancer
    b. Sarcoidosis
    c. Wegeners granulomatosis
    d. Squamous cell carcinoma
    e. Tuberculosis
    f. Coccidoidomycosis
A

a. Small cell cancer F does not normally cavitate (“distinctly rare” – StatDx) – usually central, rapid growth, early mets
4. Lung disease, which does not cavitate.
a. Small cell cancer F does not normally cavitate (“distinctly rare” – StatDx) – usually central, rapid growth, early mets
b. Sarcoidosis T cavitation of occasional nodule (atypical though)
c. Wegeners granulomatosis T cavitation occurs commonly
d. Squamous cell carcinoma T most common lung Ca to cavitate; cavitation occurs commonly in the peripheral form (although SCC more frequently)
e. Tuberculosis T cavitation is a hallmark of secondary TB
f. Coccidoidomycosis T, esp. with more chronic forms

How well did you know this?
1
Not at all
2
3
4
5
Perfectly
5
Q
  1. Most likely location of a pulmonary sequestration
    a. Right upper lobe
    b. Right middle lobe
    c. Left upper lobe
    d. Right lower lobe
    e. Left lower lobe
A

e. Left lower lobe

How well did you know this?
1
Not at all
2
3
4
5
Perfectly
6
Q
  1. Typical appearances of PCP (which is true?)
    a. Apical location (superior lobe involvement)
    b. 25% associated with pleural effusion
    c. Usually have associated hilar adenopathy
    d. Associated with CMV pneumonia
    e. A gallium scan will only become positive once the radiographic findings are clearly evident
A

d. Associated with CMV pneumonia T CMV has similar predisposition (cell-mediated immunodeficiency); Most common associated infection with PCP; also DDx for PCP; Bilateral diffuse ground-glass opacities most frequent finding
6. Typical appearances of PCP
a. Apical location (superior lobe involvement) ?T typically perihilar progressing to diffuse, often with peripheral sparing. However upper lobe distribution may occur, esp. with aerosolized pentamidine prophylaxis. Cysts, when they occur, are usually upper lobe distribution.
b. 25% associated with pleural effusion F pleural effusion very unusual (< 5%), suggests alternate Dx
c. Usually have associated hilar adenopathy F hilar adenopathy uncommon (10%), more common with other fungal or bacterial infections
d. Associated with CMV pneumonia T CMV has similar predisposition (cell-mediated immunodeficiency); Most common associated infection with PCP; also DDx for PCP; Bilateral diffuse ground-glass opacities most frequent finding
e. A gallium scan will only become positive once the radiographic findings are clearly evident F bilateral & diffuse Ga-67 uptake without mediastinal involvement prior to radiographic changes (Dahnert)

  • PCP Radiographic findings: McLoud p125/130-1, B&H p476o Initially bilateral, perihilar or diffuse, symmetric interstitial pattern – may have a finely granular, reticular/reticulonodular or ground-glass appearanceo Untreated progresses over 3-5 days to a homogeneous, diffuse alveolar consolidationo Hilar adenopathy & pleural effusion very unusual (< 5%), suggests alternate Dxo Typically diffuse perihilar with peripheral sparing – less commonly upper lobe distribution, may be assoc/ w/ aerosolised pentamidine prophylaxis
  • HRCT findings: may be positive when CXR is normalo Symmetric/bilateral with a central, perihilar or upper lobe predominance*

o Patchy or diffuse GGO*
• May result in a mosaic pattern (normal lung b/w focal GGOs)
• May progress to consolidation

o Cystic changes
• Thin or thick walls, may coalesce to form multiseptated cysts (upper lobe predominance)
• Present in 20-35% of AIDS patients, uncommon in non-AIDS patients
• May cause PTX

o Reticulation & interlobular septal thickening (resolving disease) – may produce a crazy-paving pattern (superimposed on GGO)

o Less common findings incl. focal consolidation (10% of AIDS patients), bronchiectasis or bronchiolectasis, centrilobular or diffuse nodules, pleural effusion & lymphadenopathy

How well did you know this?
1
Not at all
2
3
4
5
Perfectly
7
Q
  1. Which is most correct regarding the typical features of ARDS (LAS recall – ‘false regarding ARDS’)
    a. 10% have residual lung changes
    b. Changes are typically anterior and non-dependent (LAS – ‘residual CXR’ are in anterior non-dependent portions of lungs)
    c. CXR changes often precede clinical respiratory failure
    d. Diffuse thromboembolism is a recognized complication
A

most true A
most false B

  1. Which is most correct regarding the typical features of ARDS (LAS recall – ‘false regarding ARDS’) (StatDx)
    a. 10% have residual lung changes T – mild chronic fibrosis & low lung volumes in 10% (80% return to normal)
    b. Changes are typically anterior and non-dependent (LAS – ‘residual CXR’ are in anterior non-dependent portions of lungs) F for acute setting, T for chronic setting. Acute = Dependent intense pulmonary opacification and more nondependent ground-glass opacities (GGO) (like oil and water in a glass). Chronic = Mild reticular pattern in anterior lung (85%), residual GGO (60%), lobular hyperinflation (50%) &/or emphysema (33%).
    c. CXR changes often precede clinical respiratory failure F radiographs typically are normal for the first 12-24 hours after the acute injury, despite the presence of dyspnoea. This latent period is suggestive of ARDS

.d. Diffuse thromboembolism is a recognized complication T? = presumably have ↑ risk of thromboembolism (? Can’t find a reference)

How well did you know this?
1
Not at all
2
3
4
5
Perfectly
8
Q
  1. Patient with ascites and splenomegaly (LAS – chronic liver disease). Enhancing (vivid) round structure above the right hilum.
    a. Enlarged pulmonary vein
    b. Enlarged pulmonary artery
    c. Azygos vein
    d. Bronchogenic carcinoma
A

c. Azygos vein
Could represent an enlarged azygos vein in RHF, in the setting of CLD/portal HTN.“Azygos vein enlargement can be detected in congestive heart failure, portal hypertension, inferior vena cava thrombosis, right atrial mural thrombosis, a pulmonary embolism, congenital azygos continuation to the inferior vena cava, and the arteriovenous fistula.” Clinical Imaging July 1999.

If enlarged “hilum”, rather than “above hilum” would think of hepatopulmonary syndrome, a triad of:
• Chronic liver disease (usually cirrhosis)
• Increased alveolar-arterial oxygen gradient on room air (> 15 mmHg)
• Intrapulmonary vascular dilatation Also think of TB (increased risk in chronic liver disease)

How well did you know this?
1
Not at all
2
3
4
5
Perfectly
9
Q

40 yo man. ill defined centrilobular nodules. Non-productive cough. Thickened septal.

a. HP
b. lymphangitis
c. RA
d. asbestosis

A

A = T = hypersensitivity = centrilobular fuzzy nodules; interstitial pattern late (fibrosis)

40 yo man. ill defined centrilobular nodules. Non-productive cough. Thickened septal.

a. HP
b. lymphangitis
c. RA
d. asbestosis

A = T = hypersensitivity = centrilobular fuzzy nodules; interstitial pattern late (fibrosis)

B = F = lymphangitis = perilymphatic distribution (which incl. centrilobular, but typically perihilar; also not usually ill-defined); fissural thickening is present

C = RA = would be atypical

D = Asbestosis = no centrilobular nodules

How well did you know this?
1
Not at all
2
3
4
5
Perfectly
10
Q

smoker. 7mm nodule on CT. fisher follow up. what is it?

A

depends on what type of nodule and risk factor (low, high - only for solid lesions)

solid single
- low : f/u 6- 12mth. Then consider 18-24 mth.

  • high : f/u 6- 12 mth. Then 18- 24 mth

Partial solid- 3-6 mth.
Then every 2 years.
if solid > 6 mm is suspicious.
Ground glass - f/u 6 - 12 mth. then every 1 year. If bigger consider resection

How well did you know this?
1
Not at all
2
3
4
5
Perfectly
11
Q

which is classically associated with expansion of a lobe?

A

BAC is a cause of lobar consolidation with bulging fissures.

Also think of Klebsiella pneumonia.

How well did you know this?
1
Not at all
2
3
4
5
Perfectly
12
Q

Patient had pneumonia. Treated with antibiotics and then develops thin walled cysts with fluid level. most likely

a. pneumatoceles
b. caveatting pneumonia
c. abscess

A

A = pneumatocele = produced by virulent organisms (esp. S. aureus) – thin walled cysts filled with air or partially with fluid; thin & smooth-walled

A = pneumatocele = produced by virulent organisms (esp. S. aureus) – thin walled cysts filled with air or partially with fluid; thin & smooth-walled

B = cavitating pneumonia = thick & irregular walls, often with air-fluid level

C = abscess = thick-walled with shaggy inner lining

How well did you know this?
1
Not at all
2
3
4
5
Perfectly
13
Q

Drug related lung change. which is false

a. amiodarone and interstitial infiltrates
b. cyclosporine and mass
c. methotrexate and pneumatoceles
d. pneytoin and pleural effusions.
e. an antibiotic and hypersensitivity pneumonia or something like that

A

C = methotrexate ?F = NSIP is the most common pattern of methotrexate-induced lung disease; also common is hypersensitivity reaction; usually cause diffuse reticulonodular opacities with lower zone predominance; usually subacute allergic response

A = amiodarone T = 18% get lung toxicity, esp. chronic interstitial pneumonitis; upper zones. *LW - pneumonitis and fibrosis.

B = cyclosporine T = drug-induced lymphoproliferative disorder which may appear as solitary mass, multiple lung nodules & hilar nodes

C = methotrexate and pneumatoceles ?F = NSIP is the most common pattern of methotrexate-induced lung disease; also common is hypersensitivity reaction; usually cause diffuse reticulonodular opacities with lower zone predominance; usually subacute allergic response. *LW - hypersensitivity pneumonitis

D = phenytoin = vasculitis, drug-induced SLE; Pleural effusion is a rare complication of phenytoin use that usually occurs early in the course [Chest 2009].

E = hypersensitivity reaction T: cromolyn sodium, erythromycin, nitrofurantoin, isoniazid, penicillin, sulfonamides, bleomycin, methotrexate, procarbazine, penicillamine; features incl. Interstitial and/or alveolar opacities, patchy peripheral airspace opacities, basilar reticulonodular interstitial opacities (opacities may be fleeting)

How well did you know this?
1
Not at all
2
3
4
5
Perfectly
14
Q

Patient who you’ve done a CT guided chest biopsy on. small apical pneumothorax post procedure and 4 hrs post has not increased. Next appropriate management:

a. repeat x-ray in 24 hours
b. immediate chest tube
c. repeat car in 4 hrs
d. oxygen

A

ANS = A ? repeat CXR in 24 hours

Kandarpa V&IR p482
• Small, asymptomatic PTX, stable
o No Tx needed
• Treat PTX if: SOB, acute CP, size > 30% or continues to increase in size
o Aspiration by 18G angiocath; Place biopsy side down position; Serial CXR
o Small (11 Fr) thoracic catheter into pleural space. Can remove several hours later if air leak has ceasedJMIRO 2006In much of the published work with regard to needle lung biopsy carried out as an outpatient procedure, it is commonly cited that patients are discharged when there is no pneumothorax detected after biopsy on a chest radiograph obtained 1–3 h after biopsy. Some researchers recommend a shorter observation period of 30 min from lung biopsy to discharge of patients without pneumothorax. However, in cases complicated by pneumothorax, management thereafter has not been fixed and varies widely. Also, criteria for hospitalization after pneumothorax vary.When pneumothorax detected on CT immediately after lung biopsy is too small to provide a space for insertion of a needle for aspiration or if a detected pneumothorax completely or almost completely disappears after manual aspiration, the required observation period would be short, that is, approximately within 2 days. In such cases, the likelihood of need for a chest tube is slight and early discharge would be a strong possibility. However, in cases when the size of the pneumothorax does not change or increases despite manual aspiration, the requirement of both chest tube placement and hospitalization would be highly possible, as is suggested by the fact that all eight patients belonging to category C of the present study required chest tube placement. In addition, when the pneumothorax tends to decrease after manual aspiration but remains, chest tube placement may be required. Thus, these patients must be carefully observed until confirmation of disappearance of pneumothorax, as suggested by the three patients belonging to category B who required chest tube placement. Also supporting this suggestion is that although 23 category B cases did not require a chest tube, the period until disappearance of pneumothorax was significantly longer than category A patients or those not necessitating aspiration.

How well did you know this?
1
Not at all
2
3
4
5
Perfectly
15
Q
  1. Regarding bronchogenic cysts, which is the LEAST correct?
    a. Communicate with the bronchial tree
    b. Occur in the mediastinum more than the lung
    c. Are part of a spectrum of foregut cysts
    d. When have a thick contrast enhancing wall indicate infection
    e. High T2 signal
A

a. Communicate with the bronchial tree F Do not communicate with bronchial tree & thus do not contain air – however, may become infected, when they may gain communication with the tracheobronchial tree. Note – CCAMs do communicate with bronchial tree, while BPS does not communicate.
b. Occur in the mediastinum more than the lung T 65-90% in1.

Regarding bronchogenic cysts, which is the LEAST correct?

a. Communicate with the bronchial tree F Do not communicate with bronchial tree & thus do not contain air – however, may become infected, when they may gain communication with the tracheobronchial tree. Note – CCAMs do communicate with bronchial tree, while BPS does not communicate.
b. Occur in the mediastinum more than the lung T 65-90% in middle mediastinum; less commonly in lung parenchyma esp. lower lobes
c. Are part of a spectrum of foregut cysts T (= foregut duplication cyst)
d. When have a thick contrast enhancing wall indicate infection T wall may enhance with surrounding consolidation if infected
e. High T2 signal T

How well did you know this?
1
Not at all
2
3
4
5
Perfectly
16
Q
  1. Thoracic outlet syndrome, least likely finding on arteriography and venography.

a. Subclavian artery dissection
b. Subclavian artery aneurysm
c. Subclavian artery stenosis
d. Subclavian artery thrombosis
e. Subclavian vein thrombosis

A

a. Subclavian artery dissectionoracic outlet syndrome, least likely finding on arteriography and venography.a. Subclavian artery dissection
b. Subclavian artery aneurysm T – SCA aneurysm
c. Subclavian artery stenosis T – narrowing of SCA which is positional (abduction)
d. Subclavian artery thrombosis T – mural thrombi
e. Subclavian vein thrombosis T – may be occlusive or non-occlusive

How well did you know this?
1
Not at all
2
3
4
5
Perfectly
17
Q
  1. 25 year old is treated with antibiotics for RLL pneumonia. 1/52 later a RLL cavity with an air fluid level is demonstrated. (LAS Victoria – thick-walled)
    a. Infection of bronchogenic cyst
    b. Bronchiectasis
    c. Cavitary pneumonia
    d. Pulmonary abscess
    e. Pneumatocoele
A

**LJS - I would go cavitary pneumonia. Lots of overlap and inconsistency btw terms cavitary pneumonia, necrotising pneumonia, and lung abscess. Lung abscess often due to aspiration in immunocompromised/alcoholic etc. Seems less likely in 25yo

d. Pulmonary abscess – T rounded, focal; thick-walled with shaggy inner lining; typically caused by Staph., Klebsiella, Pseudomonas & Proteus.

25 year old is treated with antibiotics for RLL pneumonia. 1/52 later a RLL cavity with an air fluid level is demonstrated. (LAS Victoria – thick-walled) (SK, JA agree)

a. Infection of bronchogenic cyst – T 10-35% are parenchymal, usually in lower lobes medially. Presence of air indicates infection. Wall thickening may also be seen with infection (StatDx).
b. Bronchiectasis – T cystic bronchiectasis can have air-fluid levels. Least likely answer in this case though.
c. Cavitary pneumonia – T may occur due to S. aureus, gram negatives, anaerobes & TB; thick irregular walls; air-fluid level often present
d. Pulmonary abscess – T rounded, focal; thick-walled with shaggy inner lining; typically caused by Staph., Klebsiella, Pseudomonas & Proteus
e. Pneumatocoele – ?F thin-walled cyst which has thin & smooth walls & may be partially fluid-filled; more common in kids with virulent bugs (S. aureus)If thick-walled, best answer is D.

How well did you know this?
1
Not at all
2
3
4
5
Perfectly
18
Q
  1. 81 year old man presents with a pneumothorax. Which finding is MOST relevant in deciding to insert a chest drain?
    a. Complete collapse of the lung
    .b. Ipsilateral mediastinal shift.
    c. Air fluid level.
    d. Visceral pleura has a sharp outline.
    e. Lung markings are seen peripherally
A

**LJS opinion:
Hx does not state there is a haemothorax or hx of trauma/surgery to make you suspect one (and even if there were, you wouldn’t stick in a drain without knowing why they were bleeding). Common for ptx to have reactive effusion - not an indication for chest drain.
e. lung markings seen peripherally - i.e. it’s not a ptx?? If this is what is meant then this would be most relevant in deciding whether to insert a drain!
Otherwise I would say a. (unlikely to re-inflate without drain if complete collapse)

*LW:
Tricky wording, agree with LJS re peripheral lung markings imply no PTx and hence no chest drain should be inserted, while conversely the question states he presents with a PTx….
Also not enough info to decide if primary spontaneous or secondary spontaneous and clinical condition as below guidelines favour put heavy weighting on clinical stability…..

UpToDate guidelines for primary spontaneous:
–> https://www.uptodate.com/contents/image?imageKey=PULM%2F121366&topicKey=PULM%2F117232&source=see_link

  • Clinically stable patients with PSP who fail observation or aspiration, patients who are unstable due to pneumothorax, and patients with recurrent PSP should have a tube or catheter thoracostomy placed.
  • Thoracostomy is also appropriate in centers without expertise for aspiration as well as in patients with bilateral or very large pneumothoraces (eg, complete collapse), severe symptoms, concurrent hemothorax or pleural effusion necessitating drainage, complex loculated pneumothorax (unusual in PSP).

UpToDate guiedlines for secondary spontaneous:

  • Unstable patients should undergo chest tube thoracostomy.
  • Stable patients, most should be treated with chest tube as underlying lung disease more likely to increase likelihoof of aspiration failure and development of tension.

BTS 2010 guidelines:

  • decision tree is based on wheter Ptx is >2cm or breathless, neither of which are supplied in above question.
  • In a Patient > 50yrs with a spontaneous Ptx, > 3cm or breathless, chest drain recommended with admission. If less 1cm and not breathless - admit for observation.
  • Selected patients with large PTx who are asymptomatic may be managed with observation alone.
  • To add further confusion, recent NEJM paper states conservative management no inferior in outcomes….. (https://www.nejm.org/doi/full/10.1056/NEJMoa1910775)

Previous answer
c. Air fluid level. T all patients with PTX and concurrent haemothorax should undergo chest tube insertion, then thoracoscopy (UTD)

Original answer:
4. 81 year old man presents with a pneumothorax. Which finding is MOST relevant in deciding to insert a chest drain? (SK – from UTD & BTS guidelines 2003)

a. Complete collapse of the lung. ?F Degree of lung collapse not a reliable predictor – total collapse may be prevented by underlying lung disease
b. Ipsilateral mediastinal shift. ¬F contralateral mediastinal shift may indicate tension PTX, however it does not invariably indicate tension & often does not occur if on PPV
c. Air fluid level. T all patients with PTX and concurrent haemothorax should undergo chest tube insertion, then thoracoscopy (UTD)
d. Visceral pleura has a sharp outline. F a sign of PTX
e. Lung markings are seen peripherally. F

How well did you know this?
1
Not at all
2
3
4
5
Perfectly
19
Q

Bilateral perilymphatic/bronchovasuclar nodules. lymphadenopathy. no effusion. centrilobular nodules. most likely.

a. sarcoid.
b. lymphangitis carcinomatosis.

A

A = T = ticks all the boxes, classic; pleural effusion uncommon but can occur (StatDx)

B = certainly possible; 50% have an effusion

How well did you know this?
1
Not at all
2
3
4
5
Perfectly
20
Q

young woman. miliary nodular pattern with relative dense nodules. black pleura.

a. alveolar microlithiasis.
b. secondary haemosiderosis
c. sarcoid

A

A = alveolar microlithiasis = dense lungs; dense miliary calcifications with “sandstorm” appearance; “black pleura” due to small subpleural cysts; average age 35, Turkish people

B = secondary haemosiderosis = nodular interstitial thickening, often with nodular calcification (esp. longstanding mitral stenosis)

C = sarcoid = can have diffuse micronodular pattern; no black pleura though

How well did you know this?
1
Not at all
2
3
4
5
Perfectly
21
Q

man with cough not responding to antis. Subpleural shifting consolidation. most likely a. COP

A

Considerations in migratory consolidation
• Eosinophilic pneumonia
• COP
• Aspiration
• WegenerCOP
• Resembles bronchopneumonia – patchy air space consolidation
• Fails to clear with Abs! Corticosteroids may cause clearing
• Normal lung volumes

How well did you know this?
1
Not at all
2
3
4
5
Perfectly
22
Q

pneumothorax post trauma with fractured ribs. biggest reason to worry.

a. dropped lung
b. air /fluid leve
lc. other options that are definition of pneumothorax

A

***LJS opinion:
dropped lung probably means fallen lung sign - as indication of tracheobronchial injury - rare high mortality injury with high likelihood of other significant injury. I think this is a bigger reason to worry than a haemothorax (could just be bleeding intercostal vessel), might not be terrible given that we know nothing about their haemodynamic state

*LW: agree with above, if referring to fallen lung, this indicates signifcant lung injury with associated mortality, vs relatively common haemopneumothorax…fallen lung - lacerated tracheobronchial tree requires immediate surgical intevention, vs drain for large haemopneumothoraces…

previous answer
B = indicates haemopneumothorax, so needs chest tube & thoracoscopy

How well did you know this?
1
Not at all
2
3
4
5
Perfectly
23
Q

25) 25yo female. Ran marathon the previous day and presents with left sided pleuritic chest pain. Low probability VQ scan result. ED reg concerned about ?PE. Asks which imaging test is most appropriate. Which of the following is most appropriate in this clinical setting?:
i) CTPA CTCA triple rule ou
ii) CTPA
iii) Lower limb Doppler
iv) Venous Angiogram
v) Home

A

*AJL - CTPA - There is a 20% chance of having a PE with a low probability V/Q therefore CTPA is required to rule out PE.

(Previous answers)
CTPA T according to WA imaging pathways

iii) Lower limb Doppler T/F Possibly, according to UTD/PIOPED pathway, although as per UTD “Many patients with PE are likely to be missed. In one report, only 29 percent of patients with PE (determined by V/Q scan or pulmonary angiogram) had venous thrombosis detected by compression ultrasound”1)

25yo female. Ran marathon the previous day and presents with left sided pleuritic chest pain. Low probability V/Q scan result. ED reg concerned about ?PE. Asks which imaging test is most appropriate. Which of the following is most appropriate in this clinical setting?:

i) CTPA CTCA triple rule out F.
ii) CTPA T according to WA imaging pathways
iii) Lower limb Doppler T/F Possibly, according to UTD/PIOPED pathway, although as per UTD “Many patients with PE are likely to be missed. In one report, only 29 percent of patients with PE (determined by V/Q scan or pulmonary angiogram) had venous thrombosis detected by compression ultrasound”
iv) Venous Angiogram F
v) Home

How well did you know this?
1
Not at all
2
3
4
5
Perfectly
24
Q

2) 25yo female. Known asthmatic. Chest pain and SOB. Wells score 1. D-dimer –ve. ED reg concerned about ?PE. Asks which imaging test is most appropriate. Which of the following is most appropriate in this clinical setting?:
i) VQ Scan
ii) CTPA
iii) Lower limb Doppler
iv) Venous Angiogram
v) Home

A

v) Home

Negative D-dimer and low-risk (PE unlikely) clinically = PE excluded. According to WA pathways still need a CXR though

How well did you know this?
1
Not at all
2
3
4
5
Perfectly
25
Q

27) 45yo male factory worker presents to respiratory physician with 1-2 weeks of shortness of breath. Afebrile, non smoker, previously healthy. CXR shows interstitial pattern. HRCT shows poorly defined centrilobular nodules and ground glass opacity. Most likely diagnosis?
i) Hypersensitivity pneumonitis
ii) Sarcoid
iii) Silicosis
iv) Asbestosis
v) Pulmonary haemorrhage

A

i) Hypersensitivity pneumonitis T
3) 45yo male factory worker presents to respiratory physician with 1-2 weeks of shortness of breath. Afebrile, non smoker, previously healthy. CXR shows interstitial pattern. HRCT shows poorly defined centrilobular nodules and ground glass opacity. Most likely diagnosis?
i) Hypersensitivity pneumonitis T
ii) Sarcoid F perilymphatic nodules & septal thickening
iii) Silicosis F perilymphatic
iv) Asbestosis F UIP-type pattern + plaques in 80%
v) Pulmonary haemorrhage F consolidation/GGO

How well did you know this?
1
Not at all
2
3
4
5
Perfectly
26
Q

30) Post lung biopsy Xray shows PTX >50% lung volume. No shift or respiratory distress. Best management:
i) Immediate chest drain
ii) Chest drain in 4 hours
iii) Repeat CXR 4 hours
iv) Repeat CXR next day

A

i) Immediate chest drain The 2006 JMIRO article suggests needle aspiration first, then chest drain if fails. BTS (not specifically post-biopsy) says if > 2cm aspirate, then chest drain if no success. Large (>2cm width or > 50% volume)

How well did you know this?
1
Not at all
2
3
4
5
Perfectly
27
Q

31) 28 y.o female sees GP for work physical. CXR shows bilateral hilar and right mediastinal lymphadenopathy. Most likely?
i) TB
ii) Lymphoma
iii) Sarcoidosis
iv) Glandular fever
v) Bacterial Pneumonia

A

iii) Sarcoidosis T 50% asymptomatic; 1-2-3 sign (Garland’s triad)

How well did you know this?
1
Not at all
2
3
4
5
Perfectly
28
Q

Young man. rib lesion with destruction and pleural effusion. most likely.

a. askin tumour
b. fibrous dysplasia
c. neuroblastoma met

A

A = Askin tumour = PNET = young patients (M=F, 20-30 yrs); rib destruction + malignant effusion (DDx = Ewing, lymphoma, chest wall hamartoma)

A = Askin tumour = PNET = young patients (M=F, 20-30 yrs); rib destruction + malignant effusion (DDx = Ewing, lymphoma, chest wall hamartoma)

B = Fibrous dysplasia = expansile in rib; well defined borders

C = Metastases very possible, but not neuroblastoma in a young man

How well did you know this?
1
Not at all
2
3
4
5
Perfectly
29
Q

PE question. true?

a. 75% lower lobe
b. haemorrhage implies infarction
c. viable pneumocystis excludes PE?

A

A = T = 75% of infarcts occur in lower lobes(Question may belong in Path MCQs)

A = T = 75% of infarcts occur in lower lobes

B = F = haemorrhage commonly occurs WITHOUT infarction

C = F = infarcts only in 10-15%

How well did you know this?
1
Not at all
2
3
4
5
Perfectly
30
Q

TB. true?

a. primary TB more effusion than reactivation
b. primary TB is mostly in the upper lobe

A

A = T = pleural effusions more common in primary than secondary TB

Primary: - initial infection -> can be anywhere. -> predilection for upper or lower zone- imaging may be-> normal 15%-> consolidation (70% children, 90% adult),-> LN low density (90% children, 30% adult)-> pleural effusion (30-40% in adult, 10-20% in children)-> miliary 7%-> form tuberculoma (non-caveating granuloma) - mass like opacity -> Gohn’s complex (calcified tuberculoma)-> Ranke complex (gohns + calcified LN)- disease resolve if there is enough host response- completely normal 2/3- gohn 15%- tuberculoma 9%- healing : fibrosis + calcification

Post-primary TB:- location-> upper lobe posterior segment-> superior segment lower lobe- imaging-> patchy consolidation-> more likely to cavitate 50%-> LN uncommon (1/3)-> bronchial stenosis -> cause tree in bud and centrilobular nodules (10-40%)- less common appearance-> lobar consolidation- miliary TB- tuberculoma (well circumscribed mass in upper lobe) 5%

How well did you know this?
1
Not at all
2
3
4
5
Perfectly
31
Q

lateral CXR (which is false)

a. left pulmonary artery posterosuperior
b. bronchus intermedium wall < 3mm
c. right pulmonary artery and vein posterior to bronchus intermedium

A

c. right pulmonary artery and vein posterior to bronchus intermedium - false

How well did you know this?
1
Not at all
2
3
4
5
Perfectly
32
Q

36) 25yo male presents to GP for insurance medical. CXR shows linear structure paralleling right heart border thought to be scimitar vein. CXR from 2 yrs previously shows no change. Which additional finding is most likely?

i) Pulmonary sequestration
ii) Absent ipsilateral pulmonary artery
iii) Mediastinal shift away from the abnormality
iv) Hypoplastic right hemithorax with ipsilateral mediastinal shift
v) Pleural cyst

A

d. Hypoplastic right hemithorax with ipsilateral mediastinal shift T Scimitar syndrome – right lung hypoplasia, right-sided PAPVR to IVC50.

25yo (? 30 yo) male presents to GP for insurance medical (? With cough). CXR shows linear (? Curvilinear) structure paralelling right heart border thought to be scimitar vein. CXR from 2 yrs previously shows no change. Which additional finding is most likely?

a. Pulmonary sequestration F – although often assoc/ w/ anomalous systemic arterial supply to right lung base
b. Absent ipsilateral pulmonary artery F
c. Mediastinal shift away from the abnormality F shift towards
d. Hypoplastic right hemithorax with ipsilateral mediastinal shift T Scimitar syndrome – right lung hypoplasia, right-sided PAPVR to IVC
e. Pleural cyst Ff. Diaphragmatic eventration ?T – is assoc/ w/ “diaphragmatic abnormalities”

How well did you know this?
1
Not at all
2
3
4
5
Perfectly
33
Q

COP. most correct?

a. consolidation on CT
b. does not respond to steroid
c. traction bronchiectasis.
d. honeycombing

A

A = T = subpleural & peribronchovascular patchy bilateral consolidation; can be focal (persistent nodule/mass)

A = T = subpleural & peribronchovascular patchy bilateral consolidation; can be focal (persistent nodule/mass)
B = F = Corticosteroids result in rapid clearing of opacities
C = F = not listed
D = F
How well did you know this?
1
Not at all
2
3
4
5
Perfectly
34
Q
CF in a child. Typical CXR.  which is false
A. upper lobe bronchiectasis. 
b. bands of atelectasis
c. hypoinflation. 
d. confluent consolidation
A

C = F = early hyperinflation, earliest finding (StatDx) – may be reversible early & then permanent

A = T , but later finding

B = T = atelectasis & air-trapping due to mucoid plugs

C = F = early hyperinflation, earliest finding (StatDx) – may be reversible early & then permanent

D = T = esp. in setting of pneumonia (recurrent Pseudomonas, Staph & H. influenza)

How well did you know this?
1
Not at all
2
3
4
5
Perfectly
35
Q

CCAM. false.

a. most common in lingual and middle lobes.
b. pulmonary hypoplasia
c. 4 times more common in males
d. multiple cysts < 12mm
e. single cyst > 2cm

A

A = F = no lobar predilection (unlike CLE & BPS) – StatDx, DahnertNote:- no lobar predilection- if large -> can cause pulmonary hpoplasia- more common in male (radiopedia)

Association:

  • pulmonary hypoplasia
  • mixed lesion (pulmonary sequestration)- congenital heart disease
  • renal agenesis
  • cancer
  • > pleuroparenchymal blastema
  • > lung cancer
  • > rhabdomyosarcoma

Complication

  • pneumothorax
  • haematothorax
  • pyeothorax
  • cancer
  • hydrops, pulmonary hypoplasia, fetal death

type I
most common: 70% of cases 3 large cysts one or more dominant cysts: 2-10 cm in size may be surrounded by smaller cysts

type II
15-20% of cases 3 cysts are <2 cm in diameter associated with other abnormalities renal agenesis or dysgenesis pulmonary sequestration congenital cardiac anomalies

type III
~10% of cases microcysts: <5 mm in diameter typically involves an entire lobe has a poorer prognosis

type IV
unlined cyst typically affects a single lobe indistinguishable from type I on imaging

How well did you know this?
1
Not at all
2
3
4
5
Perfectly
36
Q

41) Post pneumonectomy, most concerning sign of complication
a. Rising air-fluid level
b. Mediastinal shift toward surgery side
c. Mediastinal shift away from surgery side
d. No change in position of mediastinum
e. Filling cavity
f. Decreased volume of pneumonectomy hemithorax

A

c. Mediastinal shift away pneumonectomy side ¬T concerning for bronchopleural fistula

Normal post-pneumonectomy findings
o Immediate: expanded contralateral lung, midline trachea, air-filled post-pneumonectomy space
o < 7 days: mediastinum remains midline or shifts toward surgery side
o 7 days: air-fluid level at lower 1/2 to 2/3 of hemithorax
o 2-4 months: complete filling post-pneumonectomy space with fluid
o > 4 months: opaque post-pneumonectomy space, permanent mediastinum shift to surgery side
Bronchopleural fistulao 2-13% of patientso Right > left (shorter length & less effective concealment of bronchial stump, also supplied via single bronchial artery)

o Causes:
• Early BP fistula – due to inadequate surgical technique (bronchial ischaemia) or infection of the bronchial stump
• Late BP fistula (months) – recurrent malignancyo Risk factors:
• Uncontrolled pre-op pleuropulmonary infection
• Pre-op XRT
• Steroids, malnutrition
• Trauma
• Post-op PPV
• Faulty closure of bronchial stump

o CXR
• Failure of pneumonectomy space to fill with pleural fluid
• Abrupt decrease in air-fluid level in pneumonectomy space (a drop of > 2 cm in previous location of air-fluid level
)• Contralateral shift of mediastinum following pneumonectomy
• Persistent PTX despite functioning chest tubes
• New collection of air in a previously completely opacified pneumonectomy space

How well did you know this?
1
Not at all
2
3
4
5
Perfectly
37
Q
  1. 60yo male with 20 pack year Hx of smoking. Pre-employment CXR which shows 1.5cm nodule in LLL. CT chest confirms LLL nodule and minor emphysema. No other significant findings. Full body FDG-PET is performed to assess for malignancy. PET shows no uptake in the LLL. Which of the following is the next apprioprate investigation strategy
    a. Stop, malignancy exluded
    b. Repeat CT in 3/12
    c. Dynamic contrast-enhanced CT
    d. Repeat PET 3/12
    e. Broncoscopy
A

b. Repeat CT in 3/12 T – according to WA imaging pathways, perform serial CT or biopsy For SPN larger than 1.0 cm, PET has sensitivity and specificity of 95-98% and 73-85% respectively. Tumor with low metabolic rate (low grade adenocarcinoma, BAC, carcinoid) may show minimal uptake.

How well did you know this?
1
Not at all
2
3
4
5
Perfectly
38
Q
  1. Which of the following statements regarding asbestos related chest disease is TRUE?
    a. A hemorrhagic effusion excludes the diagnosis of a benign pleural effusion
    b. Pleural effusion is usually the earliest manifestation
    c. Contrast enhancement of round atelectasis is a usual feature on CT
    d. Asbestosis commonly occurs in the absence of pleural plaques
    e. The apical and costo-phrenic angles are sites of predilication for benign pleural plaques
A

b. Pleural effusion is usually the earliest manifestation T – “Earliest asbestos-related pleural abnormality” (Dahnert); often occurs within 8-10 years of asbestos exposure. May eventually evolve into diffuse pleural thickening. StatDx: “Most common early (within 10 years of exposure) manifestation of asbestos-related pleural disease but latent period may be significantly longer”

  • *LJS - agree with b.
    https: //pubs.rsna.org/doi/full/10.1148/radiographics.22.suppl_1.g02oc10s167

c. Contrast enhancement of round atelectasis is a usual feature on CT T homogenous enhancement of atelectatic lung – can also be seen with bronchogenic carcinoma! (StatDx)
2. Which of the following statements regarding asbestos related chest disease is TRUE?
a. A hemorrhagic effusion excludes the diagnosis of a benign pleural effusion F benign effusion may be serous or blood
b. Pleural effusion is usually the earliest manifestation T – “Earliest asbestos-related pleural abnormality” (Dahnert); often occurs within 8-10 years of asbestos exposure. May eventually evolve into diffuse pleural thickening. StatDx: “Most common early (within 10 years of exposure) manifestation of asbestos-related pleural disease but latent period may be significantly longer”
c. Contrast enhancement of round atelectasis is a usual feature on CT T homogenous enhancement of atelectatic lung – can also be seen with bronchogenic carcinoma! (StatDx)
d. Asbestosis commonly occurs in the absence of pleural plaques F pleural plaques present at CT in 80% with asbestosis (StatDx)
e. The apical and costo-phrenic angles are sites of predilication for benign pleural plaques F most frequently occur on the anterior & posterolateral aspects of the costal pleura & on the diaphragmatic pleura

How well did you know this?
1
Not at all
2
3
4
5
Perfectly
39
Q
  1. With regards to cryptogenic organizing pneumonia (COP), which of the following statements is TRUE?
    a. Lung volumes are reduced in most cases
    b. There is a strong association with smoking
    c. Areas of consolidation are evident on HRCT
    d. The apices are predominantly involved
    e. Areas of fibrosis alternating with normal lung are evident on HRCT.
A

c. Areas of consolidation are evident on HRCT T

How well did you know this?
1
Not at all
2
3
4
5
Perfectly
40
Q
  1. 30yo male CXR for Ix of cough. Normal physical examination. CXR is normal except for a retrocardiac density which splays carina. No calcification or spinal lesions. MOST LIKELY explanation for the retrocardiac density is?
    a. Bronchogenic cyst
    b. Neurogenic cyst
    c. Hiatus hernia
    d. Left atrial enlargement
    e. Oesophageal tumor
A

a. Bronchogenic cyst T often subcarinal & may have mass effect
4. 30yo male CXR for Ix of cough. Normal physical examination. CXR is normal except for a retrocardiac density which splays carina. No calcification or spinal lesions. MOST LIKELY explanation for the retrocardiac density is?
a. Bronchogenic cyst T often subcarinal & may have mass effect
b. Neurogenic cyst F expect spinal changes
c. Hiatus hernia F posterior mediastinum
d. Left atrial enlargement F can cause splaying of carina, but would expect cardiomegaly (LA & LV enlargement usually, e.g. in MR)
e. Oesophageal tumor F

How well did you know this?
1
Not at all
2
3
4
5
Perfectly
41
Q
  1. CCAM, which is the most correct?
    a. 5% of all congenital lung disease
    b. The lung mass has a well-defined internal bronchial tree
    c. The mass often has a number of large air-filled cysts
    d. It most commonly appears as a homogeneous lung mass
    e. CCAM receives its blood supply from the systemic circulation
A

c. The mass often has a number of large air-filled cysts T type I lesions have cysts > 2cm
5. CCAM, which is the most correct?
a. 5% of all congenital lung disease F 25% of all congenital lung disorders, or 95% of cystic lung disease
b. The lung mass has a well defined internal bronchial tree F classically communicate with bronchial tree, however lung tissue is dysplastic
c. The mass often has a number of large air-filled cysts T type I lesions have cysts > 2cm
d. It most commonly appears as a homogeneous lung mass F variable
e. CCAM receives its blood supply from the systemic circulation F normal pulmonary vascular supply & drainage type I 50% single/multiple large cyst(s) >20mmtype II 40% multiple uniform cysts 5-12mmtype III 10% microcystic (3-5mm cysts, appears solid).

How well did you know this?
1
Not at all
2
3
4
5
Perfectly
42
Q
  1. Which of the following is NOT a significant predisposing factor for acquiring a bronchopleural fistula post pneumonectomy?
    a. Left pneumonectomy
    b. Uncontrolled pulmonary infection
    c. Pre-operative XRT
    d. Post-operative positive pressure ventilation
    e. Faulty closure of the bronchial stump
A

a. Left pneumonectomy F More common with a right sided pneumonectomy due to shorter bronchi and single bronchial artery.
6. Which of the following is NOT a significant predisposing factor for acquiring a bronchopleural fistula post pneumonectomy? (Straight from RG 06 article)
a. Left pneumonectomy F More common with a right sided pneumonectomy due to shorter bronchi and single bronchial artery

.b. Uncontrolled pulmonary infection T pre-op

c. Pre-operative XRT T
d. Post-operative positive pressure ventilation T
e. Faulty closure of the bronchial stump T

How well did you know this?
1
Not at all
2
3
4
5
Perfectly
43
Q
  1. Which of the following statements is the MOST correct description for intralobar sequestration?
    a. It does not have a complete pleural investment
    b. May occur above, within or below the diaphragm
    c. Typically has venous drainage to the pulmonary veins
    d. Usually occurs on the right side
    e. Often presents as a cavitating mass
A

c. Typically has venous drainage to the pulmonary veins T usually to inferior pulmonary vein (95% of ILS drain to pulmonary veins)
7. Which of the following statements is the MOST correct description for intralobar sequestration?
a. It does not have a complete pleural investment T no pleural covering (contained within the substance of the lung); StatDx – contiguous with normal lung & a thick fibrinous visceral pleural lining; shares visceral pleura of normal lung
b. May occur above, within or below the diaphragm F contained within the substance of the lung
c. Typically has venous drainage to the pulmonary veins T usually to inferior pulmonary vein (95% of ILS drain to pulmonary veins)
d. Usually occurs on the right side F LLL>RLL
e. Often presents as a cavitating mass T? can appear as an air-containing single or multicystic lesion (not sure if “often” though)ILS usually presents later than ELS. Patients with ILS usually present in late childhood or adolescence with recurrent pulmonary infections. ELS more apt to present in early life from resp distress / cyanosis / or assoc abnormalities. Both can be an incidental antenatal Dx too. 65% of ELS assoc with other anomalies. ILS not associated with other anomalies. ILS occurs more commonly (70%) vs ELS (30%). ELS is almost always LLL. In ILS LLL (60%) > RLL (40%). Congenital anomalies ELS>ILS.

How well did you know this?
1
Not at all
2
3
4
5
Perfectly
44
Q
  1. 10yo male, previously well. 7d Hx of fever and dry cough. CXR patchy airspace opacity at both lung bases with assoc linear atelectasis. No other abnormality. Which of the following is MOST LIKELY Dx?

a. Pneumococcal pneumonia
b. Pneumocystis pneumonia
c. Mycoplasma pneumonia
d. Staphylococcal pneumonia

A

c. Mycoplasma pneumonia – causes 30% of LRTIs in school-aged children; CAP in previously well; typically reticulonodular opacification often confined to one lobe; may evolve into patchy segmental airspace consolidation (esp. lower lobes), which may coalesce to produce lobar consolidation.

10yo male, previously well. 7d Hx of fever and dry cough. CXR patchy airspace opacity at both lung bass with assoc linear atelectasis. No other abnormality. Which of the following is MOST LIKELY Dx?

a. Pneumococcal pneumonia – most common cause of lobar pneumonia in childhood; typically homogenous consolidation with air bronchograms; lobar or sublobar
b. Pneumocystis pneumonia – unlikely without immunosuppression, perihilar
c. Mycoplasma pneumonia – causes 30% of LRTIs in school-aged children; CAP in previously well; typically reticulonodular opacification often confined to one lobe; may evolve into patchy segmental airspace consolidation (esp. lower lobes), which may coalesce to produce lobar consolidation
d. Staphylococcal pneumonia – a cause of multiple bilateral alveolar infiltrates; causes patchy bilateral opacities

How well did you know this?
1
Not at all
2
3
4
5
Perfectly
45
Q
  1. 25yo female. Ran marathon the previous day and presents with left sided pleuritic chest pain. ED reg concerned about ?PE. Asks which imaging test is most appropriate. Which of the following is most appropriate in this clinical setting?

a. CTPA CTCA triple rule out scan
b. Lower limb duplex US
c. Proceed to CTPA
d. Proceed to V/Q
e. Proceed to CTPA and CT pelvic and lower limb venography

A

c. Proceed to CTPA

Controversial
Deep Vein Thrombosis and Pulmonary Embolism in Endurance Athletes…endurance athletes are exposed to many of the physical factors contributing to DVT, experiencing repetitive microtrauma, endothelial damage, and dehydration/hemoconcentration during competition, followed by periods of inactivity, immobility, and stasis while traveling to and from and/or recovering from the athletic event. There are several published case studies regarding athletes who have experienced DVT and PE following competition or physical activity. Of interest (i.e. a possible MCQ):Chris Staniforth, a video game designer at the UK’s Leicester University, died earlier this year as a result of what was later found to be a pulmonary embolism——after he reportedly spent hours engaged in a marathon gaming session on his Xbox. Classification for PiopedHigh probabilitytwo or more large mismatched segmental defects or equivalent moderate/large defects with a normal x-rayany perfusion defect substantially larger than the radiographic abnormalityIntermediate probabilitymultiple perfusion defects with associated radiographic opacities>25% of a segment and less than two mismatched segmental perfusion defects withnormal radiographone moderate segmentalone large or two moderate segmentalone large and one moderate segmentalthree moderate segmentaltriple match: solitary moderate-large matching segmental defect with matching radiographdifficult to characterize as high probability or low probabilitysingle matched ventilation-perfusion defect with clear chest radiographthis is considered borderline low/intermediate probability, but should be classified as intermediate in most circumstances by most readers, as described in Table 5 of Gottschalk article 1Low probabilitynonsegmental defects: small effusion, blunting costophrenic angle, cardiomegaly, elevated diaphragm, ectatic aortaany perfusion defect with a substantially larger radiographic abnormalitymatched ventilation and perfusion defects with a normal chest radiographsmall subsegmental perfusion defectsNormal scanno perfusion defects

How well did you know this?
1
Not at all
2
3
4
5
Perfectly
46
Q
  1. Regarding pulmonary scintigraphy, which is TRUE?
    a. Technegas is given as a dose 5x that of macro-aggregated albumin
    b. Modified PIOPED reporting criteria includes a “normal” scan
    c. Pleural perfusion seen overlying a deeper perfusion defect is high probability
    d. Technigas examination is performed after perfusion study
    e. Pregnancy is an absolute contra-indication to V/Q scanning.
A

b. Modified PIOPED reporting criteria includes a “normal” scan T (high, intermediate, low & normal)
10. Regarding pulmonary scintigraphy, which is TRUE?
a. Technegas is given as a dose 5x that of macro-aggregated albumin ? T Technegas 900-1300 MBq, MAA 110-185 MBq
b. Modified PIOPED reporting criteria includes a “normal” scan T (high, intermediate, low & normal)
c. Pleural perfusion seen overlying a deeper perfusion defect is high probability F I think they’re talking about the “stripe sign” which has a “very low probability” of being a PE
d. Technegas examination is performed after perfusion study F the ventilation study is usually performed first, i.e. technegas before perfusion MAA
e. Pregnancy is an absolute contra-indication to V/Q scanning. F some advocate it over CTPA in pregnancy (mainly for the breast dose, fetus dose actually higher than in CTPA)

How well did you know this?
1
Not at all
2
3
4
5
Perfectly
47
Q
  1. Regarding imaging in the Immunocompromised patient, which is TRUE?

a. Low density lymphadenopathy on CT is characteristic of Kaposi’s sarcoma
B. MRI is sensitive to early changes in HIV encephalitis
c. Pneumocystic carinii pneumonia typically involves the lung bases following Rx with inhaled pentamidine
d. MAC infection, peripheral lymphadenopathy is usually more marked than abdominal lymphadenopathy
e. Toxoplasmosis lesions occur most commonly in the basal ganglia

A

e. Toxoplasmosis lesions occur most commonly in the basal ganglia T Basal ganglia, corticomedullary junction, thalamus, cerebellum most common sites (StatDx)
11. Regarding imaging in the Immunocompromised patient, which is TRUE?
a. Low density lymphadenopathy on CT is characteristic of Kaposi’s sarcoma F markedly enhancing lymphadenopathy
b. MRI is sensitive to early changes in HIV encephalopathy ?F can’t find in Dahnert, StatDx or Google.
c. Pneumocystis carinii pneumonia typically involves the lung bases following Rx with inhaled pentamidine F some have upper lobe predominance if on pentamidine (StatDx)
d. MAC infection, peripheral lymphadenopathy is usually more marked than abdominal lymphadenopathy F – thickened SB folds; mesenteric & periportal adenopathy with low-attenuation nodes
e. Toxoplasmosis lesions occur most commonly in the basal ganglia T Basal ganglia, corticomedullary junction, thalamus, cerebellum most common sites (StatDx)

How well did you know this?
1
Not at all
2
3
4
5
Perfectly
48
Q
  1. Which following drug associations is NOT an association?
    a. Cyclosporine and solitary pulmonary mass
    b. Methotrexate and pneumatoceles
    c. Amiodarone and interstitial infiltrates
    d. Phenytoin and pleural effusion
    e. Sulfasalazine and peripheral eosinophilic pneumonia
A

b. Methotrexate and pneumatoceles ?T = NSIP is the most common pattern of methotrexate-induced lung disease; also common is hypersensitivity reaction; usually cause diffuse reticulonodular opacities with lower zone predominance; usually subacute allergic response; ALTHOUGH 10% of subacute HS pneumonitis get “lung cysts” (statdx). StatDx says 10% of HSP get cysts, RG09 says ‘occasionally’ find lung cysts’ in HSP.
12. Which following drug associations is NOT an association?
a. Cyclosporine and solitary pulmonary mass T = drug-induced lymphoproliferative disorder which may appear as solitary mass, multiple lung nodules & hilar nodes
b. Methotrexate and pneumatoceles ?T = NSIP is the most common pattern of methotrexate-induced lung disease; also common is hypersensitivity reaction; usually cause diffuse reticulonodular opacities with lower zone predominance; usually subacute allergic response; ALTHOUGH 10% of subacute HS pneumonitis get “lung cysts” (statdx). StatDx says 10% of HSP get cysts, RG09 says ‘occasionally’ find lung cysts’ in HSP.
c. Amiodarone and interstitial infiltrates T = 18% get lung toxicity, esp. chronic interstitial pneumonitis; upper zones. *LW - pneumonitis and fibrosis.
d. Phenytoin and pleural effusion = ?T = vasculitis, drug-induced SLE; Pleural effusion is a rare complication of phenytoin use that usually occurs early in the course [Chest 2009].
e. Sulfasalazine and peripheral eosinophilic pneumonia T common

How well did you know this?
1
Not at all
2
3
4
5
Perfectly
49
Q
  1. Most common cause of unilateral or asymmetric pulmonary oedema
    a. COPD with fluid overload
    b. Neurogenic pulmonary oedema
    c. ARDS
    d. Excessively rapid draining of a large pleural effusion
    e. Prolonged decubitus position in congestive heart failure.
A

e. Prolonged decubitus position in congestive heart failure.Unilateral oedema with ipsilateral pathology- severe mitral valve regurgitation- positioning- re-expansion pulmonary oedema- pulmonary vein occlusion- congenital or surgical right-to-left shunt (e.g. Blalock-Taussig shunt)

How well did you know this?
1
Not at all
2
3
4
5
Perfectly
50
Q
  1. 40yo working male with 2 week Hx progressive dyspnoea. Smokes pack of cigarettes a day. Afebrile. No significant medical history. CXR shows diffuse interstitial infiltrate and HRCT shows centrilobular nodular ground glass infiltrates. MOST LIKELY Dx?
    a. Hypersensitivity pneumonitis
    b. Asbestosis
    c. RB-ILD
    d. Pulmonary hemorrhage
A

c. RB-ILD T “Centrilobular ground-glass opacities in the upper lobes”
14. 40yo working male with 2 week Hx progressive dyspnoea. Smokes pack of cigarettes a day. Afebrile. No significant medical history. CXR shows diffuse interstitial infiltrate and HRCT shows centrilobular nodular ground glass infiltrates. MOST LIKELY Dx?
a. Hypersensitivity pneumonitis F smoking protects from hypersensitivity pneumonitis (StatDx)
b. Asbestosis F 20-40 year latency period (i.e. too young); UIP pattern
c. RB-ILD T “Centrilobular ground-glass opacities in the upper lobes”
d. Pulmonary hemorrhage – F diffuse haemorrhage usually acute onset with diffuse bilateral GGO & consolidation

How well did you know this?
1
Not at all
2
3
4
5
Perfectly
51
Q
  1. Enlargement of lobe of lung?
    a. Alveolar cell carcinoma
    b. Pulmonary sequestration
    c. Bronchiectasis
    d. Meliodiosis
    e. Radiation pneumonitis
A

a. Alveolar cell carcinoma T Lobar consolidation may cause lobar expansion with bulging fissures (StatDx) – now called adenocarcinoma in-situ (previously BAC)
15. Enlargement of lobe of lung?
a. Alveolar cell carcinoma T Lobar consolidation may cause lobar expansion with bulging fissures (StatDx) – now called adenocarcinoma in-situ (previously BAC)
b. Pulmonary sequestration F
c. Bronchiectasis F
d. Meliodiosis F Burkholderia pseudomallei is a fungus which can present as pneumonia; can cause lobar consolidation, can’t find lobar expansion (ERJ September 1, 2003 vol. 22 no. 3 542-550)
e. Radiation pneumonitis F Pulmonary consolidation with sharply defined linear or curvilinear interface

How well did you know this?
1
Not at all
2
3
4
5
Perfectly
52
Q
  1. Not characteristic of UIP
    a. Basal predominance
    b. Nodules
    c. Honeycombing
    d. Reticular pattern
    e. Traction bronchiectasis
A

b. Nodules T not a feature of IPF & most other causes of UIP pattern, but can occur in UIP secondary to HSP

How well did you know this?
1
Not at all
2
3
4
5
Perfectly
53
Q
  1. Lateral CXR, most correct?
    a. Left pulmonary artery forms the superior and anterior aspects of hilar shadow
    b. Anterior part of hilar complex is isodense with heart
    c. RUL bronchus lies above LUL / Left main bronchus
    d. Composite shadow of right pulmonary artery and right and left pulmonary veins lies behind bronchus intermedius
    e. Posterior wall of bronchus intermedius should be 5mm thick
A

c. RUL bronchus lies above LUL / Left main bronchus T see below
17. Lateral CXR, most correct?
a. Left pulmonary artery forms the superior and anterior aspects of hilar shadow F superior & posterior
b. Anterior part of hilar complex is isodense with heart ?F
c. RUL bronchus lies above LUL / Left main bronchus T see below
d. Composite shadow of right pulmonary artery and right and left pulmonary veins lies behind bronchus intermedius F lies anterior to BI
e. Posterior wall of bronchus intermedius should be 5mm thick F normally < 3mm

How well did you know this?
1
Not at all
2
3
4
5
Perfectly
54
Q
  1. Which of the following is not a typical radiographic feature of CF in children?

a. CXR abnormality at birth
b. Hyperinflation of lungs
c. Upper lobe predominant bronchiectasis
d. Hilar lymphadenopathy
e. Apical cystic airspaces

A

a. CXR abnormality at birth F lungs normal at birth
18. Which of the following is not a typical radiographic feature of CF in children
a. CXR abnormality at birth F lungs normal at birth
b. Hyperinflation of lungs T
c. Upper lobe predominant bronchiectasis T
d. Hilar lymphadenopathy T – recurrent infection, superimposed on PAH
e. Apical cystic airspaces T upper lobe predominance

How well did you know this?
1
Not at all
2
3
4
5
Perfectly
55
Q

60.A 24 year old athlete presents with left pleuritic chest pain after running a marathon yesterday. VQ scan was reported as low probability of PE. CXR showed no abnormality. The ED medical staff phone you for advice regarding the next most appropriate step and you advise:

  1. No further imaging required
  2. CTPA
  3. Repeat the VQ scan in 1 week
  4. Ultrasound both legs
  5. CTPA and lower limb venography
A
  1. CTPA - T Possibly, according to WA imaging pathways
  2. A 24 year old athlete presents with left pleuritic chest pain after running a marathon yesterday. VQ scan was reported as low probability of PE. CXR showed no abnormality. The ED medical staff phone you for advice regarding the next most appropriate step and you advise: (SK)
  3. No further imaging required - F
  4. CTPA - T Possibly, according to WA imaging pathways
  5. Repeat the VQ scan in 1 week - F – you want another low probability result?
  6. Ultrasound both legs - T/F Possibly, according to UTD/PIOPED pathway, although as per UTD “Many patients with PE are likely to be missed. In one report, only 29 percent of patients with PE (determined by V/Q scan or pulmonary angiogram) had venous thrombosis detected by compression ultrasound”
  7. CTPA and lower limb venography - F -
How well did you know this?
1
Not at all
2
3
4
5
Perfectly
56
Q
  1. Regarding the thymus, which is most correct?
  2. An increase in size post chemotherapy is indicative of disease recurrence
  3. Hyperplasia of the thymus is the most common cause of a mediastinal mass in adolescence
  4. Thymolipoma compresses vessels
  5. Thymoma spreads by direct extension along pleural reflections
A
  1. Thymoma spreads by direct extension along pleural reflections – T/F – True if question specified ‘invasive’ thymoma which is locally invasive, spread by contiguity along pleural reflections, extension along aorta reaching posterior mediastinum/crus/retroperitoneum. [Dahnert pg537]
  2. Regarding the thymus, which is most correct? (GC/SK) RG 2010 article
  3. An increase in size post chemotherapy is indicative of disease recurrence - F - rebound hyperplasia may occur post chemotherapy (atrophies during chemo, then grows back to its original/larger size after cessation). However, a mediastinal mass arising in adolescents and adults after treatment for cancer is suspicious of relapse [Eur J Pediatr 1992]. Distinguishing a prominent but normal thymus in young pts, and thymic hyperplasia from lymphomatous involvement is problematic; the use of chemical shift MRI and PET have been described [Thymus RG 2006]. The thymus may show striking FDG avidity in rebound hyperplasia (RG 2010].
  4. Hyperplasia of the thymus is the most common cause of a mediastinal mass in adolescence – T/F - Normal thymus reaches its maximum (absolute) weight in puberty and subsequently undergoes involution. Thymic hyperplasia is the most common cause of an anterior mediastinal mass in adolescent (D p536). HOWEVER “Lymphoma is the most common cause of an anterior mediastinal mass in children and the second most common cause of an anterior mediastinal mass in adults [RG 2010]. AND “Lymphoma is the most common anterior mediastinal mass in children” [Radiology assistant]
  5. Thymolipoma compresses vessels – T/F - may drape around heart (enlarged CTR, or apparent elevated hemidiaphragm); no compression/invasion of adjacent structures; fatty mass with fibrous septa. HOWEVER RG 2010 states they are pliable, but about 50% cause mass effect without invading neighbouring structures – also although mostly asymptomatic, can present with compression-related symptoms
  6. Thymoma spreads by direct extension along pleural reflections – T/F – True if question specified ‘invasive’ thymoma which is locally invasive, spread by contiguity along pleural reflections, extension along aorta reaching posterior mediastinum/crus/retroperitoneum. [Dahnert pg537] BEST ANSWER – LIKELY NO. 4
How well did you know this?
1
Not at all
2
3
4
5
Perfectly
57
Q
  1. 65 year old male construction worker. Dyspnoea and hilar mass with calcifications. The most likely explanation is
  2. Asbestosis
  3. Silicosis
  4. Sarcoidosis
  5. Tuberculosis
A

2.Silicosis - T - given Hx of construction worker. Complicated silicosis (= PMF) - large opacities in hilar region resulting from nodule coalescence. The large opacities gradually migrate toward hilum. CT features of PMF are focal soft-tissue masses, often with irregular or ill-defined margins and calcifications, surrounded by emphysematous change. Simple silicosis = multiple nodular opacities that are well defined and uniform in shape and attenuation and that range from 1-10mm in diameter. Upper lobe and posterior portion of lung predominance. Calcification of nodules seen on CXR in 10-20%. 3.

65 year old male construction worker. Dyspnoea and hilar mass with calcifications. The most likely explanation is (TW)

  1. Asbestosis - F - potential exposure, but would have calcified plaques, and round atelectasis peripheral and not calcified.
  2. Silicosis - T - given Hx of construction worker. Complicated silicosis (= PMF) - large opacities in hilar region resulting from nodule coalescence. The large opacities gradually migrate toward hilum. CT features of PMF are focal soft-tissue masses, often with irregular or ill-defined margins and calcifications, surrounded by emphysematous change. Simple silicosis = multiple nodular opacities that are well defined and uniform in shape and attenuation and that range from 1-10mm in diameter. Upper lobe and posterior portion of lung predominance. Calcification of nodules seen on CXR in 10-20%.
  3. Sarcoidosis
  4. Tuberculosis
How well did you know this?
1
Not at all
2
3
4
5
Perfectly
58
Q

63.Which of the following is the most correct regarding primary tuberculosis (more than secondary form)?

  1. Upper lobe cavity
  2. Effusion more in primary
  3. Absence of lymphadenopathy
  4. Consolidation in posterior segment of upper lobe
A
  1. Effusion more in primary - T - pleural effusions occur most often in primary TB which occurs in about 25% patients (effusion is often sole manifestation of TB and usually manifests 3-7 months after initial exposure). Effusion is usually unilateral. In postprimary TB approx 18% of patients have effusion (usually small and associated with parenchymal disease; effusion typically septated and can remain stable in size for many years).
  2. Which of the following is the most correct regarding primary tuberculosis (more than secondary form)? (TW)
  3. Upper lobe cavity - F - predominance in lower and middle lobes, especially in adults, typically adjacent to interlobar fissure.
  4. Effusion more in primary - T - pleural effusions occur most often in primary TB which occurs in about 25% patients (effusion is often sole manifestation of TB and usually manifests 3-7 months after initial exposure). Effusion is usually unilateral. In postprimary TB approx 18% of patients have effusion (usually small and associated with parenchymal disease; effusion typically septated and can remain stable in size for many years).
  5. Absence of lymphadenopathy - F - distinguishing feature of postprimary TB vs primary.
  6. Consolidation in posterior segment of upper lobe - F - earliest finding in parenchymal TB in postprimary disease is patchy, poorly defined consolidation, particularly in the apical and posterior segments of the upper lobes. Majority of cases more than one pulmonary segment is involved, with bilateral disease in 1/3 to 2/3rds.
How well did you know this?
1
Not at all
2
3
4
5
Perfectly
59
Q

64.Which is the most correct regarding a lateral CXR?

  1. Right upper lobe bronchus higher than left upper lobe bronchus
  2. Left pulmonary artery forms anterior and superior aspect of hilum
  3. Right pulmonary artery passes behind the bronchus intermedius
  4. The posterior border of the bronchus intermedius should measure <5mm
A
  1. Right upper lobe bronchus higher than left upper lobe bronchus - T - see diagram
  2. Which is the most correct regarding a lateral CXR? (TW)
  3. Right upper lobe bronchus higher than left upper lobe bronchus - T - see diagram
  4. Left pulmonary artery forms anterior and superior aspect of hilum - F superior & posterior
  5. Right pulmonary artery passes behind the bronchus intermedius - F – RPA/conglomerate vascular opacity is anterior to bronchus intermedius on lat radiograph.
  6. The posterior border of the bronchus intermedius should measure <5mm – T/F - posterior wall of the bronchus should be < 3.0mm in thickness.
60
Q

65.Post pneumonectomy CXR. The most concerning feature would be

  1. Increased air fluid level
  2. Shift of mediastinum to the ipsilateral side
  3. Shift of mediastinum to the opposite side
  4. No mediastinal shift
  5. Absent post pneumonectomy space
A
  1. Shift of mediastinum to the opposite side - T – RG 2006 “a shift of the mediastinum to the side opposite the postpneumonectomy space” coupled with a 2cm drop in A-F level is consistent with bronchopleural fistula
  2. Post pneumonectomy CXR. The most concerning feature would be (TW)
  3. Increased air fluid level - F – TW gave this as true, but it depends how you interpret the question – the key sign of bronchopleural fistula is a DROP in the air-fluid level of ≥ 1.5cm, with a shift of the mediastinum to the side opposite the post-pneumonectomy space – so if they mean increased “height” of the A-F level answer is false, if they mean increase in amount of air in cavity then answer is true.Other radiographic features suggestive of BPF = failure of postpneumonectomy space to fill; persistent or progressive PTX despite adequate tube drainage; progressive subcutaneous or mediastinal emphysema; a 2-cm drop in the air-fluid level, with a shift of the mediastinum to the contralateral side (<1.5cm drop may be positional, >1.5cm is more indicative of fistula).
  4. Shift of mediastinum to the ipsilateral side - F - an expected finding as fluid / gas progressively resorbs from postpneumonectomy site.
  5. Shift of mediastinum to the opposite side - T – RG 2006 “a shift of the mediastinum to the side opposite the postpneumonectomy space” coupled with a 2cm drop in A-F level is consistent with bronchopleural fistula
  6. No mediastinal shift - F
  7. Absent post pneumonectomy space - F - not sure what they’re meaning here ?maybe surgeon forgot to take out lung

Normal/expected post-pneumonectomy findings:o Immediate: expanded contralateral lung, midline trachea, air-filled post-pneumonectomy space

o < 7 days: mediastinum remains midline or shifts toward surgery side

o 7 days: air-fluid level at lower 1/2 to 2/3 of hemithoraxo 2-4 months: complete filling post-pneumonectomy space with fluid

o > 4 months: opaque post-pneumonectomy space, permanent mediastinum shift to surgery side

o Right pneumonectomy: heart shifts to right & posteriorly; left lung herniates anterior to heart

o Left pneumonectomy: heart shifts to left; right lung herniates anterior or posterior to heartBronchopleural fistula

o 2-13% of patientso Right > left (shorter length & less effective concealment of bronchial stump, also supplied via single bronchial artery)

o Causes:
• Early BP fistula – due to inadequate surgical technique (bronchial ischaemia) or infection of the bronchial stump
• Late BP fistula (months) – recurrent malignancyo Risk factors:
• Uncontrolled pre-op pleuropulmonary infection
• Pre-op XRT
• Steroids, malnutrition
• Trauma
• Post-op PPV
• Faulty closure of bronchial stump

o CXR
• Failure of pneumonectomy space to fill with pleural fluid
• Abrupt decrease in air-fluid level in pneumonectomy space (a drop of > 2 cm in previous location of air-fluid level)
• Contralateral shift of mediastinum following pneumonectomy
• Persistent PTX despite functioning chest tubes
• New collection of air in a previously completely opacified pneumonectomy space
o Xenon ventilation study – demonstrates xenon activity in the pneumonectomy space
o Mortality 20-70%, from aspiration pneumonia and ARDS

61
Q

66.Most common cause for unilateral pulmonary oedema:

  1. COPD with volume overload
  2. Decubitus positioning with heart failure
  3. Rapid drainage of pleural effusion
  4. ARDS
  5. Raised intracranial pressure
A
  1. Decubitus positioning with heart failure - T - the most common cause of ipsilateral edema is the decubitus position (Thoracic Imaging).
  2. Most common cause for unilateral pulmonary oedema: (TW)
  3. COPD with volume overload - F - less common (if big bullae / emphysema more on one side) or bilateral.
  4. Decubitus positioning with heart failure - T - the most common cause of ipsilateral edema is the decubitus position (Thoracic Imaging).
  5. Rapid drainage of pleural effusion - F - less common
  6. ARDS - F - bilateral
  7. Raised intracranial pressure - F - neurogenic pulmonary oedema -typically present within minutes to hours of a severe CNS insult. Bilateral.

Hydrostatic edema usually is bilateral and symmetric. In patients with edema, variations in the distribution of the edema fluid may result from variations in blood flow or hydrostatic pressure.

Unilateral oedema may be seen due to ipsilateral or contralateral abnormalities in blood flow or hydrostatic pressure:
Unilateral oedema assoc with ipsilateral abnormalities: decubitus position, reexpansion oedema; pulmonary vein occlusion; congenital or acquired systemic-pulmoanry shunt; papillary muscle ruture and mitral regurgitation.

Unilateral oedema with contralteral abnormalities: pulmonary artery occlusion; hypoplastic pulmonary artery interruption of pulmonary artery; Swyer-James; unilateral emphysema or bullae; transient atelectasis.

62
Q

67.25yo male presents to GP for insurance medical. CXR shows linear structure paralelling right heart border thought to be aberrant vessel. CXR from 2 yrs previously shows no change. Which additional finding is most likely?

  1. Small lung
  2. Pleural cyst
  3. Absent ipsilateral pulmonary artery
  4. Mediastinal shift away from the the abnormality
  5. Pulmonary sequestration
A
  1. Small lung - T - Scimitar syndrome (aka hypogenetic lung sx) = right lung hypoplasia, dextroversion of heart, prominent RA, scimitar vein in right medial costophrenic sulcus. The latter is a curved anomalous venous trunk connecting to the IVC, no right pulm veins entering LA (ie. infracardiac PAPVR). Presentation depends on size of L-R shunt; often asymptomatic in older child (incidental finding on CXR). [Pocket paeds]
  2. 25yo male presents to GP for insurance medical. CXR shows linear structure paralelling right heart border thought to be aberrant vessel. CXR from 2 yrs previously shows no change. Which additional finding is most likely? (GC)
  3. Small lung - T - Scimitar syndrome (aka hypogenetic lung sx) = right lung hypoplasia, dextroversion of heart, prominent RA, scimitar vein in right medial costophrenic sulcus. The latter is a curved anomalous venous trunk connecting to the IVC, no right pulm veins entering LA (ie. infracardiac PAPVR). Presentation depends on size of L-R shunt; often asymptomatic in older child (incidental finding on CXR). [Pocket paeds]
  4. Pleural cyst - F
  5. Absent ipsilateral pulmonary artery - F
  6. Mediastinal shift away from the the abnormality - F
  7. Pulmonary sequestration - F
63
Q

69.45yo male factory worker presents to respiratory physician with 1-2 weeks of shortness of breath. Afebrile, non smoker, previously healthy. CXR shows interstitial pattern. HRCT shows poorly defined centrilobular nodules and ground glass opacity. Most likely diagnosis?

  1. Hypersensitivity pneumonitis
  2. Sarcoid
  3. Silicosis
  4. Asbestosis
  5. Pulmonary haemorrhage
A
  1. Hypersensitivity pneumonitis - T - acute/subacute stage - small, ill-defined centrilobular nodules, bilateral airspace consolidation, GGO (patchy distribution).
  2. 45yo male factory worker presents to respiratory physician with 1-2 weeks of shortness of breath. Afebrile, non smoker, previously healthy. CXR shows interstitial pattern. HRCT shows poorly defined centrilobular nodules and ground glass opacity. Most likely diagnosis? (GC)
  3. Hypersensitivity pneumonitis - T - acute/subacute stage - small, ill-defined centrilobular nodules, bilateral airspace consolidation, GGO (patchy distribution).
  4. Sarcoid - F - perilymphatic nodules; +/-GGO/consolidation/crazy paving; adenopathy in 85%. Upper zone predilection.
  5. Silicosis - F - acute silicoproteinosis (heavy exposure, fulminant course) - patchy GGO, crazy paving. Chronic simple silicosis (10-20yrs exp) - upper +posterior long zones, perilymphatic nodules +/- GGO, fibrous bands, traction/honeycombing.
  6. Asbestosis - F - bilateral, lower posterior bases, subpleural septal thickening, patchy GGO.
  7. Pulmonary haemorrhage - F - GGO/consolidation; would have haemoptysis (cf. SOB). [Dahnert]
64
Q

70.10yo patient presents with 7 day history of dry cough, febrile. Chest x ray shows bilateral patchy lower lobe airspace opacification with associated subsegmental atelectasis. Most likely diagnosis?

  1. Mycoplasma pneumonia
  2. Pneumococcus
  3. TB
  4. Staphylococcus aureus
A
  1. Mycoplasma pneumonia - T - most common in 5-20yo.; dry cough, low fever, malaise; pulmonary infiltrates show significant lag time - focal reticular infiltrate (incl. atelectasis), alveolar infiltrates (patchy inhomogeneous unilateral airspace consolidation in segmental LL in 50%, bilateral in 10-40%); small pleural effusions & hilar LNs in some.
  2. 10yo patient presents with 7 day history of dry cough, febrile. Chest x ray shows bilateral patchy lower lobe airspace opacification with associated subsegmental atelectasis. Most likely diagnosis? (GC)
  3. Mycoplasma pneumonia - T - most common in 5-20yo.; dry cough, low fever, malaise; pulmonary infiltrates show significant lag time - focal reticular infiltrate (incl. atelectasis), alveolar infiltrates (patchy inhomogeneous unilateral airspace consolidation in segmental LL in 50%, bilateral in 10-40%); small pleural effusions & hilar LNs in some.
  4. Pneumococcus - F - uncommon in children, causes round pneumonia. More common in elderly, debilitated, alcoholics, CHF, COPD etc; usually involves one lobe only (predilection for LL, posterior segment of UL) but may have a bronchopneumonia-like pattern.
  5. TB - F - primary TB is most common form in infants and kids, 90% are asymptomatic; one or more areas of homogeneous dense well-defined airspace consolidation 1-7cm in middle/lower/anterior segment upper lobes. Reactivation of focus acquired in childhood (post primary TB) usually occurs in adolescence/adulthood; 85% occurs in apicoposterior UL, 10% in superior segment LL.
  6. Staphylococcus aureus - F - in kids causes rapidly developing lobar/multilobar consolidation, pleural effusion/empyema (90%) and pneumatocoele (40-60%). [Dahnert]
65
Q

72.30 yo male presents with cough. Chest x ray is normal except for a retrocardiac opacity which splays the carina. No spinal abnormality. Most likely?

  1. Enlarged left atrium
  2. Hiatus hernia
  3. Neurogenic cyst
  4. Bronchogenic cyst
A
  1. Bronchogenic cyst – T (SK) most often subcarinal & may have mass effect – can displace trachea or bronchi (see pic below); large bronchogenic cysts may be assoc/ w/ symptoms because of compression of adjacent structures; F TW/GC: 52% pericarinal, 9% retrocardiac; may show tracheal compression; usually asymptomatic (stridor, dysphagia may occur) [Dahnert]. The true incidence of BCs is difficult to determine because of the asymptomatic nature of most BCs. Average incidence being 20 cases over a 20y period.
  2. 30 yo male presents with cough. Chest x ray is normal except for a retrocardiac opacity which splays the carina. No spinal abnormality. Most likely? (SK/GC,TW)
  3. Enlarged left atrium - F (SK) can cause splaying of carina, but would expect cardiomegaly (LA & LV enlargement usually, e.g. in MR)T (GC, TW) - mitral stenosis: hoarseness can develop from compression of the left recurrent laryngeal nerve against the pulmonary artery by an enlarged left atrium. Also, compression of bronchi by the enlarged left atrium can cause a persistent cough. Haemoptysis may occur and is usually not fatal. It occurs because of the rupture of thin, dilated bronchial veins due to left atrial hypertension. [eMedicine]
  4. Hiatus hernia - F - posterior mediastinum; retrocardiac mass +/- air-fluid level; usually positioned to left of spine; however, larger hernias (esp. incarcerated) may extend beyond the cardiac confines or mimic cardiomegaly. Cough and/or wheeze may occur in the setting of GORD resulting from the aspiration of gastric contents. [eMedicine]
  5. Neurogenic cyst - F - expect spinal changes; develop at a location in which the dorsal foregut and the primitive notochord are in close relationship. Symptoms are more commonly observed in kids, whereby airway compression may produce persistent cough, SOB, stridor. A triad consisting of the presence of a mediastinal mass, symptoms referable to the airway, and the presence of a vertebral anomaly has been observed in approximately 70% of pediatric patients with neurenteric cysts. Communication of a neurenteric cyst with the spinal canal has been reported. Patients with this added abnormality may develop neurologic symptoms, including gait abnormalities, back pain, and motor or sensory deficits. [eMedicine]

4.Bronchogenic cyst – T (SK) most often subcarinal & may have mass effect – can displace trachea or bronchi (see pic below); large bronchogenic cysts may be assoc/ w/ symptoms because of compression of adjacent structures; F TW/GC:
52% pericarinal, 9% retrocardiac; may show tracheal compression; usually asymptomatic (stridor, dysphagia may occur) [Dahnert]. The true incidence of BCs is difficult to determine because of the asymptomatic nature of most BCs. Average incidence being 20 cases over a 20y period.

5.Oesophageal tumour F

66
Q
  1. 55 yo male lifelong smoker cough and 2 wks of haemoptysis. CT shows 3cm peripheral RUL mass with some surrounding emphysema and small ipsilateral effusion. What is the best method of obtaining tissue for diagnosis?
  2. Surgical lobectomy
  3. Sputum cytology
  4. Percutaneous biopsy
  5. Transbronchial biopsy
  6. Aspiration of effusion
A
  1. Percutaneous biopsy - T - but surrounding emphysema means an increased risk of PTX. Good because peripheral location would mean it is unlikely to get a diagnostic yield of cells in sputum/bronchial washing/brushing/BAL or TBFNA. Sensitivity 89%, specificity 96%. Contraindications: COAD, uncontrolled coughing, bleeding disorders, severe PAH, AVM, suspected hydatid cyst. Most common Cx is PTX - usually minor and detectable on CXR (10% require chest tube drainage). Transient haemoptysis occurs in 5-10% of cases.
  2. 55 yo male lifelong smoker cough and 2 wks of haemoptysis. CT shows 3cm peripheral RUL mass with some surrounding emphysema and small ipsilateral effusion. What is the best method of obtaining tissue for diagnosis? (GC)
  3. Surgical lobectomy - F - lobectomy is appropriate for dx of centrally located masses. VATS with wedge-resection would be feasible in this case (peripherally located mass 1-5cm in size).
  4. Sputum cytology - F - low yield in this case; good for Ca arising from proximal tracheobronchial tree.
  5. Percutaneous biopsy - T - but surrounding emphysema means an increased risk of PTX. Good because peripheral location would mean it is unlikely to get a diagnostic yield of cells in sputum/bronchial washing/brushing/BAL or TBFNA. Sensitivity 89%, specificity 96%. Contraindications: COAD, uncontrolled coughing, bleeding disorders, severe PAH, AVM, suspected hydatid cyst. Most common Cx is PTX - usually minor and detectable on CXR (10% require chest tube drainage). Transient haemoptysis occurs in 5-10% of cases.
  6. Transbronchial biopsy - F - peripheral location means a dx yield of 30-50% (less if lesion <2cm). Addition of BAL, brushings and bronchoscopic aspiration increase the yield to ~75%.
  7. Aspiration of effusion - F - low yield. [Mel Fuentes reg talk; General thoracic surgery, Shields 2004; Essentials of lung tumour cytology, Nguyen 2008]
67
Q

74.What is true regarding PICC lines?

  1. Cephalic vein is the preffered access point
  2. Tip position should be in brachiocephalic vein
  3. Ideal device for TPN
  4. Should be placed above elbow
A
  1. Should be placed above elbow - T - generally at or above elbow crease. Try to prevent movement of catheter and kinking with elbow movement.
  2. What is true regarding PICC lines? (GC)
  3. Cephalic vein is the preffered access point - F - overall, the basilic vein is the optimal site for PICC, followed by the brachial veins and the cephalic vein.
  4. Tip position should be in brachiocephalic vein - F - at the junction of SVC and right atrium.
  5. Ideal device for TPN - F - parenteral nutrition solutions should be infused into a large central vein to reduce the risk of intimal damage from the catheter and infusate. Often a tunneled central venous catheter is used. Although PICC line administered TPN is described. TPN solution ismore viscous and larger lumen required.
  6. Should be placed above elbow - T - generally at or above elbow crease. Try to prevent movement of catheter and kinking with elbow movement.
68
Q
  1. CCAM most likely
  2. at least one microcyst
  3. solid homogenous
  4. 5% congenital lung disease
  5. supplied by systemic vessel
  6. intact bronchial tree
A

**LJS opinion - bronchial tree is not intact as it is characterised by dysplastic bronchial tissue. It communicates with an intact bronchial tree but the bronchial tree within the lesion is not intact.
I think 1. is most correct - even the macrocystic ones have some microcysts (together with multiple large cysts), so all have at least microcysts +/- macrocysts in type 1 CPAM

*LW: agree with LJS re option 5: by definition CPAM is harmatomatous lesion, so normal tissue arranaged in a haphazard way, so bronchial tree although may, is usually not intact with in the lesion, but the lesion can communicate with the normal pulmonary bronchial tree, although this is unusual (per Radiographics). Agree with option 1, in that even type 1 (large cysts), commonly have small surrounding cysts.

Previous answer
5.intact bronchial tree - T - intralobar mass of disorganised pulmonary tissue communicating with the bronchial tree. [Dahnert]

16.CCAM most likely: (GC)1.at least one microcyst - F -
type I single/multiple large cyst(s) >20mm;
type II multiple uniform cysts 5-12mm;
type III microcystic (3-5mm cysts, appears solid).

  1. solid homogenous - F - type III may appear as a homogeneously hyperechoic mass on antenatal US but only accounts for 10%. Type I (50%) +type II (40%) are macrocystic with air-fluid or fluid filled cysts on CT/CXR.
  2. 5% congenital lung disease - F - 25% of congenital lung disorders; 95% of congenital cystic lung lesions.
  3. supplied by systemic vessel - F - normal vascular supply/drainage (cf. sequestration).
  4. intact bronchial tree - T - intralobar mass of disorganised pulmonary tissue communicating with the bronchial tree. [Dahnert]
69
Q
  1. Which is least likely to cause cystic air spaces?
  2. LAM
  3. Hodgkins lymphoma
  4. Mycoplasma pneumonia
  5. Sjogren Syndrome
A
  • LW:
    1. LAM: commonly
    2. Hodgkins lymphoma: relatively uncommon, although possible: A study by Filly et al. [24] analyzing the incidence of intraparenchymal involvement in HL and NHL revealed nodules and cysts in only 11.6% of Hodgkin’s disease patients and an even lower percent in the NHL population.
    3. Mycoplasma pneumonia: not commonly reported in major rad references causing cysts / pneumatoceles, so this would be my prefered option of being least likely to cause cystic air spaces.
    4. Sjogren Syndrome: Via LIP, so relatively a common cause of cystic lung disease (still a rare condition).

Previous answer
3.Mycoplasma pneumonia - F - commonest cause of non-bacterial CAP with a mild course. Focal reticular interstitial infiltrate. Parahilar peribronchial opacification. Atelectasis. Alveolar infiltrates (D).

  1. Which is least likely to cause cystic air spaces? (TW)
  2. LAM - T - Pathologically - LAM is characterised by non-neoplastic hamartomatous proliferation of atypical muscle cells that leads to progressive cystic destruction of lung parenchyma. Diffuse involvement of entire lung. No predominance for any one area (DI).
  3. Hodgkins lymphoma - T - Lung involvement by HD is seen in 10% at time of presentation. Almost always assoc with mediastinal adenopathy. When lung is involved, discrete, single, or multiple well-defined or ill-defined, large or small nodules or mass-like lesions,, or localized areas of air-space consolidation assoc with air bronchograms may bee seen. These can cavitate, with thin or thick walls (TI)
  4. Mycoplasma pneumonia - F - commonest cause of non-bacterial CAP with a mild course. Focal reticular interstitial infiltrate. Parahilar peribronchial opacification. Atelectasis. Alveolar infiltrates (D).
  5. Sjogren Syndrome - T - (lymphocytic interstitial pneumonia) Older women cf LAM. 1/3 have thin walled cysts (DI) Added option 4.
70
Q
  1. Which is atypical for bronchiolitis in children?
  2. Lymphadenopathy
  3. Perihilar markings
  4. Hyperinflation
  5. Lobar consolidation
A
  1. Lobar consolidation - F - this would be case if secondary bacterial infection. Lobar pneumonia typically caused by organisms such as strep pneumonia and klebsiella.
  2. Which is atypical for bronchiolitis in children? (TW)
  3. Lymphadenopathy - T - viral infection. Can be seen with LRTI in kids , a less worrying sign c.f. in adults (Statdx).
  4. Perihilar markings - T - radiographic abnormalities of bronchiolitis are variable and nonspecific. Hyperinflation from air trapping, peribronchial thickening. Pathchy atelectasis with volume loss may result from airway narrowing and mucus plugging.
  5. Hyperinflation - T - air-trapping is common in patients with acute bronchiolitis.
  6. Lobar consolidation - F - this would be case if secondary bacterial infection. Lobar pneumonia typically caused by organisms such as strep pneumonia and klebsiella. Bronchiolitis is a LRTI that primarily affects the small airways. Typically caused by viral infection with RSV the most common cause. Less common causes include parainfluenza, human metapneumovirus, influenza, adenovirus, rhinovirus, coronavirus, and human bocavirus. Added option 3.
71
Q

79.Which is the least use in investigating possible foreign body aspiration?

  1. Inspiratory and expiratory films
  2. PA and lateral CXR
  3. Lateral decubitus
  4. Fluoroscopy
  5. Bronchoscopy
A
  1. PA and lateral CXR - F - plain radiographic evaluation may or may not be helpful in establishing the Dx of FBA depending on whether the FB is opaque (9% cases). Most objects aspirated by children are radiolucent and ar not detected with standard radiographs unless aspiration is accompanied by airway obstruction or other complications.
  2. Which is the least use in investigating possible foreign body aspiration? (TW)
  3. Inspiratory and expiratory films - T - in children with a suggestive presentation and a normal inspiratory chest radiograph, an expiratory chest radiograph or fluoroscopy may be helpful (looking for mediastinal shift away from the lung field containing the foreign body, or the post obstructive hyperinflation secondary to air trapping distal to the FB.
  4. PA and lateral CXR - F - plain radiographic evaluation may or may not be helpful in establishing the Dx of FBA depending on whether the FB is opaque (9% cases). Most objects aspirated by children are radiolucent and ar not detected with standard radiographs unless aspiration is accompanied by airway obstruction or other complications.
  5. Lateral decubitus - T - this may simulate expiratory radiographs in young children in whom it is difficult to obtain expiratory radiographs. Note however a retrospective study in 2007 (Assefa D; Pediatr Emerg Care) concluded “Decubitus chest radiographs, at least as routinely performed and interpreted, seem to add little to the evaluation of young children with suspected FB aspiration” - because… - see ans 5. This arguement could probably be applied to ans 1, 2, 3, and 4…. all end in bronch if there is suspicion.
  6. Fluoroscopy - T - see ans 1.
  7. Bronchoscopy - T - the tracheobronchial tree should be examined in all cases of suspected foreign body aspiration. Morbidity and mortality may be increased if bronchoscopic evaulation is delayed. (UTD)
72
Q

80.34 y.o non-smoking male with dyspnoea, ?asthma. Respiratory function tests normal. HRCT shows ground glass opacity with centrilobular nodules. Most likely?

  1. Sarcoidosis
  2. Hypersensitivity pneumonitis
  3. Lymphangitis
  4. NSIP
A
  1. Hypersensitivity pneumonitis - T - acute/subacute stage - small, ill-defined centrilobular nodules, bilateral airspace consolidation, ground-glass opacities (patchy distribution). Individuals must be susceptible (allergic response), most dust-exposed individuals (90%) have no response.
  2. 34 y.o non-smoking male with dyspnoea, ?asthma. Respiratory function tests normal. HRCT shows ground glass opacity with centrilobular nodules. Most likely? (TW)
  3. Sarcoidosis - F - peribronchovascular distribution, subpleural nodules, adenopathy.
  4. Hypersensitivity pneumonitis - T - acute/subacute stage - small, ill-defined centrilobular nodules, bilateral airspace consolidation, ground-glass opacities (patchy distribution). Individuals must be susceptible (allergic response), most dust-exposed individuals (90%) have no response.
  5. Lymphangitis - F - can cause centrilobular nodules and perilymphatic nodules, less likely given age and Hx. Nodular or beaded septal thickening. Patchy ground-glass and airspace opacities. (DI)
  6. NSIP - F - HRCT bilateral symmetrical ground-glass opacities. Nodules not a feature. Gradual onset. 40-50yo.
73
Q

81.28 y.o male with cough. CT shows hilar and mediastinal lymphadenopathy. Most likely?

  1. Sarcoidosis
  2. Lymphoma
  3. Glandular fever
  4. HIV
  5. Industrial lung disease
A
  1. Sarcoidosis - T - 10-40/100000. age onset usually 20-40y. 80% lymphadenopathy (bilateral hilar / paratracheal). Most common signs/symptoms - fatigue, malaise, wt loss, fever, night sweats, dyspnoea, dry cough. 1-2-3 pattern: (1) right paratracheal, (2) right hilar, (3) left hilar node enlargement. (4) sometimes added for AP window. (TW)
  2. Lymphoma - F - 8-15/100000 (NHL). NHL 4x more common than HD, however HD tends to have greater thoracic involvement. Affects all ages with median age 55yo. Extranodal NHL can have hilar and mediastinal adenopathy with DDx of sarcoidosis. Hilar adenopathy in 25% of pt with HD and 10% of NHL.
  3. Glandular fever - F - often begins with malaise, headache, low grade fever before development of more specific signs of tonsillitis and/or pharyngitis, cervical lymph node enlargement (lymphadenopathy characteristically is symmetric an dinvolves the posterio cervical chain more than the anterior) (UTD). Can also cause hilar lymphadenopathy, but less frequent. Incidence 38 / 100000. Typically patients present between 15-25yo.
  4. HIV - F - less likely hod
  5. Industrial lung disease - F - unless put to the mines at a very young age (chronic simple silicosis 10-20y of dust exposure before appearance of roentgenographic abnormality).
74
Q

84.Hot tub lung, subacute stage HRCT, best answer

  1. Bronchiectasis
  2. Centrilobular nodules and ground glass opacification
  3. Dyspnoea
  4. Air space consolidation
  5. Air trapping
A
  1. Ground glass centrilobular nodules - T - HRCT is often first performed in the subacute stage of HP, weeks to months following 1st exposure (Hot tub lung - mycobacteria). Typical HRCTR findings include patchy GGO (50-70%) or small ill-defined centrilobular nodules of GGO (40-70%). The presence of GGO nodules or pathcy GGO in a patient with known antigen exposure and typical symptoms is usually diagnostic in clinical practice (TI).
  2. Hot tub lung, subacute stage HRCT, best answer (TW)
  3. Bronchiectasis - F - may see in chronic stage (hypersensitivty pneumonitis) along with fibrosis and honeycombing
  4. Ground glass centrilobular nodules - T - HRCT is often first performed in the subacute stage of HP, weeks to months following 1st exposure (Hot tub lung - mycobacteria). Typical HRCTR findings include patchy GGO (50-70%) or small ill-defined centrilobular nodules of GGO (40-70%). The presence of GGO nodules or pathcy GGO in a patient with known antigen exposure and typical symptoms is usually diagnostic in clinical practice (TI).
  5. Dyspnoea - F - acute stage, presenting symptom… unless continued exposure
  6. Air-space consolidation - F - acute stage: alveolar filling by neutrophlic inflammatory exudate and pulmonary oedema or haemorrhage due to diffuse alveolar damage (in acute stage). HRCT in this stage may show bilatera air-space consolidation and small, ill-defined CL nodular nodules.
  7. Air trapping – F? - potentially could be true. Often see mosiac perfusion on inspiratory HRCT or airt trapping on expiratory HRCT in subacute stage.
75
Q

85.Which is not part of the Well’s criteria for PE

  1. Tachycardia
  2. Haemoptysis
  3. Can’t think of better diagnosis
  4. Hypotensive
A

4.Hypotensive - F Well’s criteria (plus score): - clinically suspected DVT (3); - alternative Dx is less likely than PE (3);- tachycardia (1.5); - immobilisation/surgery in previous 4 weeks (1.5); - history of DVT or PE (1.5); - haemoptysis (1); - malignancy (treatment for within 6 months, palliatve) (1).

76
Q
  1. Not a feature of UIP/IPF?
  2. IPF can be Dx without Bx
  3. Good response to steroids
  4. Apico-basal gradient
  5. Cigarette smoking is a risk factor
A
  1. Good response to steroids - F - usually, patients do not respond to high-dose corticosteroid therapy (and due to significant side effects of steroids, this may actually be contraindicated). Steroids have effect in NSIP. With acute exacerabations of IPF a combination therapy of cyclosporin A and steroids may be efficacious.
  2. Not a feature of UIP/IPF? (TW)
  3. IPF can be Dx without Bx - T - the ATS ERS has defined 8 major and minor criteria for diagnosis of IPF in the absence of surgical lung Bx.
  4. Good response to steroids - F - usually, patients do not respond to high-dose corticosteroid therapy (and due to significant side effects of steroids, this may actually be contraindicated). Steroids have effect in NSIP. With acute exacerabations of IPF a combination therapy of cyclosporin A and steroids may be efficacious.
  5. Apico-basal gradient - T - in the typical patient with UIP, the disease is most extensive on the most basal section of the HRCT (subpleural).
  6. Cigarette smoking is a risk factor - T - *AJL - Robbins says smoking is the most important environmental risk factor and increases the risk of IPF several fold.
77
Q

87.Not a feature of COP?

  1. Non smokers
  2. May be seen in patients with collagen vascular diseases
  3. Subpleural sparing
  4. Centrilobular nodules with ground-glass opacities and bronchial wall thickening
A
  1. Centrilobular nodules with ground-glass opacities and bronchial wall thickening - F - these are features of RB-ILD
  2. Not a feature of COP? (TW)
  3. Non smokers - T - no association with cigarette smoking, and most patients are nonsmokers or ex-smokers
  4. May be seen in patients with collagen vascular diseases - T - COP may occur in a wide variety of entities, notably in collagen vascular diseases and in infectious and drug-induced lung diseases.
    * AJL - COP = cryptogenic OP = idiopathic. If a patient has CVD or relevant infectious or drug history then the organising pneumonia is not idopathic. I think this answer is therefore false.
  5. Subpleural sparing - T - lung abnormalities show a characteristic peripheral or peribronchial distribution, and the lower lung lobes more frequently involved. In some cases, the outermost subpleural area is spared. SK – F – typical distribution subpleural &/or peribronchovascular
    * LW: COP shows a peribronchovascular and subpleural distribution, and rarely can show sub pleural sparing.

4.Centrilobular nodules with ground-glass opacities and bronchial wall thickening - F - these are features of RB-ILD SK – T – can have ill-defined
peribronnchiolar/centrilobular nodes & bronchial wall thickening (Mueller – see below)
*LW: Centrilobular nodules are rare and infrequent in COP, but may be seen. Opacity is more commonly consolidation rather than ground glass. Thus, this is the favoured answer, being not a feature of COP.

78
Q

88.Positive Gallium scan of lungs, which is false:

  1. Lymphoma. If lymphoma in the lungs
  2. Sarcoid.
  3. PCP.
  4. Cardiac amyloid.
  5. Kaposi’s sarcoma
A
  1. Kaposi’s sarcoma. F - not Ga-avid - thereby differentiates from infection and lymphoma in AIDS patients.
  2. Positive Gallium scan of lungs, which is false: (GC)
  3. Lymphoma. T - helps determine residual disease versus inactive fibrosis. Uptake in Hodgkin’s disease (nodular sclerosing and mixed cellularity) - persistent uptake during and after treatment is a poor prognostic sign and suggests a high risk for relapse. Also may have uptake in NHL (intermediate and high grade).
  4. Sarcoid. T - pulmonary uptake correlates with disease activity and reponse to therapy, reported to be up to 97% sensitive for detection of active sarcoidosis when considering both pulmonary and extrapulmonary sites. A grading system (0-4) may be used to try to quantitate - 0 is normal, 1 equivocal, 2 - 4 represent abnormal activity that is less than/equal to/greater than hepatic activity. [see diagram in B&H, pg 1442]
  5. PCP. T - intense diffuse uptake is indicative of PCP, particularly useful in HIV patients.
  6. Cardiac amyloid. T - intense and diffuse uptake
  7. Kaposi’s sarcoma. F - not Ga-avid - thereby differentiates from infection and lymphoma in AIDS patients. Thallium is positive in KS and lymphoma. Thallium scanning is performed first, as downscatter from the high Ga energies into the thallium window will degrade the images. Bound to iron transport proteins (transferrin, ferritin, lactoferrin). Localises in inflammations and tumours because of uptake in leukocytes, siderophores (LMW chelates produced by bacteria), and abnormal vascular permeability at these sites (increased delivery of transferrin).

Pulmonary uptake of Ga-67 can be seen at 24hrs in normal studies. Imaging is performed at 48hrs, by which time normal lung tissue has cleared. Other causes of Ga-67 lung uptake: -

Focal: primary pulmonary malignancy, benign disorders (granuloma, abscess, pneumonia, silicosis) -

Diffuse/multifocal: infection (TB, PCP, CMV), inflammation (sarcoid, ILD & IPF, pneumoconiosis, lymphangitis, radiation pneumonitis), drugs (bleomycin, amiodarone), contrast lymphaniography. [Teaching Atlas of Nuc Med; Kaposi Sarcoma RG 2006; Dahnert & Primer] Changed Qu from “is seen in”

79
Q

Gallium 67 scanning, which is false:

  1. Bowel excretion occurs
  2. Gallium is taken up by appendicular skeleton.
  3. Half life of 6hrs.
  4. Imaging performed after 24hrs.
A
  1. Half life of 6hrs. F - 78 hrs
  2. In Gallium 67 scanning, which is false: (GC)
  3. Bowel excretion occurs T - GIT excretion via hepatobiliary and colonic mucosa. Enemas and laxatives promote clearing of bowel activity. Also excreted via urinary tract and body fluids (including breast milk).
  4. Gallium is taken up by appendicular skeleton. T - major radiation is to bone marrow (with uptake most pronounced in lumbar spine and SI joints), colon, spleen and liver. Also uptake in
  5. Half life of 6hrs. F - 78 hrs
  6. Imaging performed after 24hrs. T - 48-72hrs post IV administration (up to 7 days); earlier if looking for abscess (6-24hrs).
80
Q
  1. The following statements regarding thoracic neoplasms in AIDS patients are true:
  2. Positive gallium scan distinguishes Lymphoma from Kaposi’s sarcoma.
  3. Kaposi’s sarcoma is typically peripheral in distribution.
  4. Lymph nodes involved with Kaposi’s sarcoma don’t enhance strongly with IV contrast.
  5. Pleural effusions occur in about 5% of patients with pulmonary Kaposi sarcoma.
  6. There is a strong association of AIDS with Hodgkin’s lymphoma.
A
  1. Positive gallium scan distinguishes Lymphoma from Kaposi’s sarcoma. T - thallium is positive in both, but KS is not Ga-avid.
  2. The following statements regarding thoracic neoplasms in AIDS patients are true: (GC)
  3. Positive gallium scan distinguishes Lymphoma from Kaposi’s sarcoma. T - thallium is positive in both, but KS is not Ga-avid.
  4. Kaposi’s sarcoma is typically peripheral in distribution. F - Mid to lower zones, peribronchial/perivascular axial interstitium.
  5. Lymph nodes involved with Kaposi’s sarcoma don’t enhance strongly with IV contrast. F - image below shows enhancing nodes in pt with disseminated KS.
  6. Pleural effusions occur in about 5% of patients with pulmonary Kaposi sarcoma. F - occur in 1/3 to 2/3 of patients. The presence of pleural collections has been associated with shortened survival times.
  7. There is a strong association of AIDS with Hodgkin’s lymphoma. F - generally high grade lymphomas of B-cell origin; thought to be related to immunosuppression, although cause remains unclear. Incidence of NHL in AIDS patients was 60-200 times higher than in a matched HIV-negative population prior to the introduction of HAART, and the RR was even greater for primary cerebral lymphoma. Hodgkin’s disease is occasionally seen.
81
Q

91.In Kaposi’s Sarcoma, which is true:

  1. Bronchovascular distribution.
  2. Hypodense nodes.
  3. Uptake of Gallium
A
  1. Bronchovascular distribution. T - Mid to lower zones, peribronchial/perivascular axial interstitium. Characteristically, bilateral and symmetric ill-defined nodules in a peribronchovascular distribution (flame-shaped lesions), usually >1cm diameter +/- “halo sign”. Other common findings: peribronchovascular/interlobular septal thickening, fissural nodularity, pleural collections, and hilar or mediastinal adenopathies.
  2. In Kaposi’s Sarcoma, which is true: (GC)
  3. Bronchovascular distribution. T - Mid to lower zones, peribronchial/perivascular axial interstitium. Characteristically, bilateral and symmetric ill-defined nodules in a peribronchovascular distribution (flame-shaped lesions), usually >1cm diameter +/- “halo sign”. Other common findings: peribronchovascular/interlobular septal thickening, fissural nodularity, pleural collections, and hilar or mediastinal adenopathies.
  4. Hypodense nodes. F - strongly enhancing nodes are seen.
  5. Uptake of Gallium. F - not Ga-avid - thereby differentiates from infection and lymphoma in AIDS patients. Thallium is positive in KS and lymphoma. [Kaposi Sarcoma RG 2006]
82
Q

92.Regarding Interstitial pneumonitis, which is most correct:

  1. NSIP typically honey-combing and basal distribution.
  2. AIP seen in immunodeficent.
  3. COOP increased in smokers.
  4. UIP is commonest in women over 50yrs.
  5. LIP typically bilateral ground-glass opacity with scattered cysts.
A

5.LIP typically bilateral ground-glass opacity with scattered cysts. T - thin-walled perivascular cysts occur within the lung parenchyma throughout the mid zones and presumably result from air trapping due to peribronchiolar cellular infiltration. GGO is related to diffuse interstitial inflammation. Associated with Sjorgren’s syndrome and HIV.

83
Q
  1. Features of TB, which is true:
  2. Homogenous enhancement of nodes.
  3. Dense ascites.
  4. Lung infiltrate in primary TB is most commonly in the upper lobe.
  5. Early joint space destruction is a characteristic feature of TB arthritis.
  6. The most common signs of pericardial involvement are effusion and calcification
A
  1. Dense ascites. T - wet type peritonitis (most common) features large amounts of free or loculated ascites, which is usually hyperattenuating (20-45 HU) due to its high protein and cellular content. Other types are fibrotic-type perionitis (large omental and mesenteric cakelike masses with matting of bowel loops), and dry-type peritonitis (mesenteric thickening, fibrous adhesions and caseous nodule
  2. Features of TB: (GC)
  3. Homogenous enhancement of nodes. F - any nodes >2cm in diameter generally have a low-attenuation centre (secondary to necrosis), and hyperattenuating enhancing rims
  4. Dense ascites. T - wet type peritonitis (most common) features large amounts of free or loculated ascites, which is usually hyperattenuating (20-45 HU) due to its high protein and cellular content. Other types are fibrotic-type perionitis (large omental and mesenteric cakelike masses with matting of bowel loops), and dry-type peritonitis (mesenteric thickening, fibrous adhesions and caseous nodule).
  5. Lung infiltrate in primary TB is most commonly in the upper lobe. F - predominance in lower and middle lobes, esp in adults.
  6. Early joint space destruction is a characteristic feature of TB arthritis. F - insidious onset, with relative preservation of joint space, and relative absence of bony proliferation or periosteal reaction.
  7. The most common signs of pericardial involvement are effusion and calcification F - pericardial thickening (>3mm in adults) is seen in the majority of cases. Most paitents have distension of the IVC >3cm, pleural effusions (typically bilateral), and deformities of the IV septum. Less than 20% have pericardial effusions or develop localised pericardial calcificaiton. [TB review RG 2007; Dahnert]
84
Q

94.In adult primary thoracic tuberculosis

:1.Lymphadenopathy does not occur as an isolated finding.

  1. Miliary tuberculosis occurs in approximately 25%.
  2. Presence of calcification is a reliable feature of inactive disease.
  3. Pleural effusion usually occurs within 6 weeks of the initial exposure
  4. Ghon focus is the typical sequelae of parenchymal disease in immunocompetent patients
A
  1. Presence of calcification is a reliable feature of inactive disease. T - with treatment, there is usually slower resolution of lymphadenopathy than of the parenchymal disease, and nodal calcifications may develop. Calcification usually occurs at least 6 mths after the initial infection. (TW) Active vs inactive TB: In general, one must have prior radiographics for comparision to determine disease activity, and radiographic activity should be stable for 6 months or more before suggesting disease is not active. An exception to this rule is that one may confidently assume that calcified lung nodules represent inactive disease (TI).
  2. In adult primary thoracic tuberculosis: (GC)
  3. Lymphadenopathy does not occur as an isolated finding. F - although usually associated with other manifestations of TB, it can be the sole radiographic feature, a finding that is more common in infants and decreases in frequency with age.
  4. Miliary tuberculosis occurs in approximately 25%. F - up to 7%; massive haematogenous dissemination of organisms any time after primary infection. Usually seen in elderly, infants and imunocompromised.
  5. Presence of calcification is a reliable feature of inactive disease. T - with treatment, there is usually slower resolution of lymphadenopathy than of the parenchymal disease, and nodal calcifications may develop. Calcification usually occurs at least 6 mths after the initial infection. (TW) Active vs inactive TB: In general, one must have prior radiographics for comparision to determine disease activity, and radiographic activity should be stable for 6 months or more before suggesting disease is not active. An exception to this rule is that one may confidently assume that calcified lung nodules represent inactive disease (TI).
  6. Pleural effusion usually occurs within 6 weeks of the initial exposure F - most commonly occurs 3-7 mths after exposure. Usually unilateral, complications (empyema, fistula, bone erosion) are rare.
  7. Ghon focus is the typical sequelae of parenchymal disease in immunocompetent patients F - approx 2/3 of cases will resolve without sequelae (although this can take up to 2 years). A radiologic scar is seen in the remaining 1/3 = can calcify in up to 15% of cases. Rhanke complex = calcified hilar nodes + Ghon focus (22%). Simon focus = healed site of primary infection in the lung apex.
85
Q

95.Air trapping is not seen in:

  1. EAA.
  2. Lower lobe apical segment on HRCT is normal.
  3. Kaposi’s sarcoma.
  4. Brochiolitis obliterans.
A
  1. Kaposi’s sarcoma - Characteristically, bilateral and symmetric ill-defined nodules in a peribronchovascular distribution (flame-shaped lesions), usually >1cm diameter +/- “halo sign”. Other common findings: peribronchovascular/interlobular septal thickening, fissural nodularity, pleural collections, and hilar or mediastinal adenopathies.
  2. Air trapping is not seen in: (GC)
  3. EAA. - aka hypersensitivity pneumonitis. Allergic lung disease caused by inhalation of antigens contained in a variety of organic dusts. Usually presents as either GGO in mosaic distribution, or centrilobular nodules of ground glass density. Areas of decreased perfusion + mosaic perfusion seen in 86% of subacute EAA; focal air trapping / diffuse emphysema seen in chronic EAA.
  4. Lower lobe apical segment on HRCT is normal. -
  5. Kaposi’s sarcoma - Characteristically, bilateral and symmetric ill-defined nodules in a peribronchovascular distribution (flame-shaped lesions), usually >1cm diameter +/- “halo sign”. Other common findings: peribronchovascular/interlobular septal thickening, fissural nodularity, pleural collections, and hilar or mediastinal adenopathies.
  6. Brochiolitis obliterans. - mosaic perfusion of lobular air trapping in 85-100%. [Radiology Assistant; Dahnert] Mueller: “In many normal subjects, areas of air-trapping are visible on expiratory scans (Figs. 2-22 and 2-23); in these regions, lung does not increase normally in attenuation and appears relatively lucent. This appearance is most typically seen in the superior segments of the lower lobes or in the anterior middle lobe or lingula, or it involves individual pulmonary lobules, particularly in the lower lobes [45,57]; it is limited to a small proportion of lung volume.”
86
Q

96.Man with 7mm RLL nodule on CT with spiculated margins what is the next appropriate test?

  1. Contrast enhancement pattern.
  2. Biopsy
  3. PET.
  4. Repeat scan in 3 months.
  5. Review old films if stable unlikely to be malignant.
A

*AJL
Fleischner guidelines have changed since this question was written.
Follow up would now be 6-12 month CT and the 18-24 month repeat.
I think the correct answer is to first review old films. The lesion may have been present and stable for years. Depends on the wording of the question.

*LW:
Spiculation, although suggestive of malignancy is not diagnostic, and conversely non spiculated nodules can still be cancer, so hence treat as a solitary pulmonary nodule of 7mm.
Solid: scan at 6-12months, consider 18-24 repeat.
GGO: Ct at 6-12months.
Part solid: CT at 3-6months, then if persistent and solid remains < 6mm, annual CT 5 years.
only mentions tissue bioipsy as first option if solid nodule > 8mm.
–> so.. although review old films is what we do, they havent mentioned how old the films are, while if they mentioned 5 year old films and no change, then this would equal stability, while conversely if it was only 3 months prior, this is not stability.
—> Thus, my preferred option, if available for modern F society guidelines, would be repeat CT in 6-12months, assuming not a part solid nodule.

Previous answer:
4.Repeat scan in 3 months - T - according to the FS guidelines - low or high risk patient, with inital f/u at 3-6mo in high-risk. Note that the article in Radiology 2006 says “the presence of spiculation has a predictive value for malignancy of approx 90% and should prompt an aggressive work-up. While an irregular margin is indicative of malignancy, it can occasionally be seen in granulomatous disease, lipoid pneumonia, organizing pneumonia, and progressive massive fibrosis”.

  1. Man with 7mm RLL nodule on CT with spiculated margins what is the next appropriate test? (TW)
  2. Contrast enhancement pattern - F
  3. PET scan - F - small
  4. Biopsy - F - however could be true. Radiology article: “FNAB has sensitivity of 86% and specificty of 98.8% in diagnosis of malignancy, however in nodules of 5-7mm in diameter, sensitivity is only 50%. For nodules that have clinical and imaging features of malignancy, a tissue sample is required”.
  5. Repeat scan in 3 months - T - according to the FS guidelines - low or high risk patient, with inital f/u at 3-6mo in high-risk. Note that the article in Radiology 2006 says “the presence of spiculation has a predictive value for malignancy of approx 90% and should prompt an aggressive work-up. While an irregular margin is indicative of malignancy, it can occasionally be seen in granulomatous disease, lipoid pneumonia, organizing pneumonia, and progressive massive fibrosis”.
  6. review old films if stable unlikely to be malignant - F - would follow this up 3-6, thn 9-12 and 24mo.
87
Q

Lung cancer staging

A

TNM system
T: primary tumour
Tx: primary tumour cannot be assessed or tumour proven by the presence of malignant cells in sputum or bronchial washings but not visualised by imaging or bronchoscopy
T0: no evidence of a primary tumour
Tis: carcinoma in situ - tumour measuring 3 cm or less and has no invasive component at histopathology
T1: tumour measuring 3 cm or less in greatest dimension surrounded by lung or visceral pleura without bronchoscopic evidence of invasion more proximal than the lobar bronchus (i.e. not in the main bronchus)
T1a(mi): minimally invasive adenocarcinomatumour has an invasive component measuring 5 mm or less at histopathology
T1a ss: superficial spreading tumour in central airways (spreading tumour of any size but confined to the tracheal or bronchial wall)

T1a: tumour ≤1 cm in greatest dimension
T1b: tumour >1 cm but ≤2 cm in greatest dimension
T1c: tumour >2 cm but ≤3 cm in greatest dimension

T2: tumour >3 cm but ≤5 cm or tumour with any of the following features:

  • involves the main bronchus regardless of distance from the carina but without the involvement of the carina
  • invades visceral pleura
  • associated with atelectasis or obstructive pneumonitis that extends to the hilar region
  • involving part or all of the lung

T2a: tumour >3 cm but ≤4 cm in greatest dimension
T2b: tumour >4 cm but ≤5 cm in greatest dimension

T3: tumour >5 cm but ≤7 cm in greatest dimension or:

  • associated with separate tumour nodule(s) in the same lobe as the primary tumour, or
  • directly invades any of the following structures:
  • chest wall (including the parietal pleura and superior sulcus)
  • phrenic nerve
  • parietal pericardium

T4: tumour >7 cm in greatest dimension or associated with separate tumour nodule(s) in a different ipsilateral lobe than that of the primary tumour or invades any of the following structure:

  • diaphragm
  • mediastinum
  • heart
  • great vessels
  • trachea
  • recurrent laryngeal nerve
  • oesophagus
  • vertebral body
  • carina.

It is recommended that solid and non-solid lesions should be measured on the image that shows the greatest tumour dimension (on axial, coronal, or sagittal planes). Although those lesions that are part solid should be measured on both their largest average diameter and the largest diameter of the solid component, only the solid component measurement is to be used for staging directions 3. Also, the solid component of subsolid lesions should be performed on a lung or intermediate window rather than mediastinal window 3.

N: regional lymph node involvement
Nx: regional lymph nodes cannot be assessed
N0: no regional lymph node metastasis
N1: metastasis in ipsilateral peribronchial and/or ipsilateral hilar lymph nodes and intrapulmonary nodes, including involvement by direct extension
N2: metastasis in ipsilateral mediastinal and/or subcarinal lymph node(s)
N3: metastasis in contralateral mediastinal, contralateral hilar, ipsilateral or contralateral scalene, or supraclavicular lymph node(s)
Please note that has been no changes in the nodal involvement staging since the 7th edition of the IASLC.PET-CT plays an important role in staging the nodal disease. FDG uptake higher than the blood pool is suspicious, and uptake higher than the liver it is highly concerning for nodal metastases. Endobronchial biopsy of an FDG-avid node is recommended to confirm the disease highest pathologic stage 4.

M: distant metastasis
M0: no distant metastasis
M1: distant metastasis present

M1a: separate tumour nodule(s) in a contralateral lobe; tumour with pleural or pericardial nodule(s) or malignant pleural or pericardial effusions
M1b: single extrathoracic metastasis, involving a single organ or a single distant (nonregional) nodea single extrathoracic metastasis has a better survival and different treatment choices, reason why it has now been staged separately
M1c: multiple extrathoracic metastases in one or more organs

88
Q
  1. Posterior mediastinal mass, which is least likely to be central:
  2. Dermoid.
  3. Varices.
  4. Pancreatic pseudocyst
  5. Neuroblastoma.
  6. Lymphangioma.
A
  1. Neuroblastoma. - malignant tumour of primitive neural crest cells; most common posterior mediastinal mass in young children. P paravertebral in location; normally the right inferior paravertebral soft tissues are not visualised in kids - any visualisation is abnormal; may see associated intercostal widening or rib erosion.
  2. Posterior mediastinal mass, which is least likely to be central: (GC)
  3. Dermoid. - mediastinum is most common extragonadal location for germ cell tumours; derive from germ cell rest remnants in the mediastinum. Mature teratomas occur in the anterior superior mediastinum; posterior mediastinum is in 3-8%.
  4. Varices. - oesophageal varices.
  5. Pancreatic pseudocyst. - extension into the mediastinum is a rare but serious complication of acute pancreatitis; dissects along planes of least resistance and may extend through potential communication sites (ie. aortic or oesophageal hiatus) into the posterior medisatinum (more rarely through the foramen of Morgagni into the anterior mediastinum).
  6. Neuroblastoma. - malignant tumour of primitive neural crest cells; most common posterior mediastinal mass in young children. P paravertebral in location; normally the right inferior paravertebral soft tissues are not visualised in kids - any visualisation is abnormal; may see associated intercostal widening or rib erosion.
  7. Lymphangioma - collection of dilated and proliferated lymph vessels; neck (80%), mediastinum, axilla, extremity. May have chylous / pleural effusion, may have lytic lesion in contiguous skeleton.
89
Q
  1. Known associations, which is false
  2. Bleomycin and pulmonary fibrosis
  3. Methotrexate and pneumatocoeles
  4. Amiodarone and organizing pneumonia
  5. Cocaine and intra alveolar hemorrhage
A
  1. Methotrexate and pneumatocoeles - F - most common pulmonary toxicity assoc with MTX is hypersensitivity pneumonitis (UTD).
  2. Known associations, which is false (TW)
  3. Bleomycin and pulmonary fibrosis - T - chemotherapeutic antibiotic isolated from a strain of Streptomyces verticillus. Rx of a variety of malignancies (SCC heand and neck, cervix, oesophagus, germ cell tumors, HD, NHL). Has potential for life-threatening pulmonary fibrosis in up to 10% of patients receiving the drug (UTD). LW - penumonitis and fibrosis.
  4. Methotrexate and pneumatocoeles - F - most common pulmonary toxicity assoc with MTX is hypersensitivity pneumonitis (UTD).
  5. Amiodarone and organizing pneumonia - T - an organizing pneumonia with or without bronchioloitis obliterans (BOOP pattern) is seen in approx 25% of cases of amiodarone pulmonary toxicity (UTD).
  6. Cocaine and intra alveolar hemorrhage - T - hemoptysis has been reported in 6-26% of crack users and is considered to be either (a) secondary to rupture of bronchial or tracheal submucosal blood vessels, or (b) parenchymal in origin, arising from injury to the alveolar-capillary membrane (Radiographics 2007) Added options 1,3. Changed Qu to ‘which is false’.
90
Q

102.Bronchiolitis obliterans - Constrictive bronchiolitis, which of the following are false: (T/F)

  1. Upper lobe predominance
  2. Mosaic perfusion
  3. CF
  4. Methotrexate is a treatment for BO
  5. Lung transplant, heart-lung, and BM particularly susceptible
A

false 1 and 4

  1. Bronchiolitis obliterans, which of the following are false: (T/F) (TW)
  2. Upper lobe predominance - F - bilateral scattered heterogeneous and homogeneous opacities (typically peripheral in distribution; equally distributed between upper and lower lobes).
  3. Mosaic perfusion - T - of lobular air trapping (85-100%). Small airways problems: path - irreversible fibrosis of small airway walls with narrowing / obliteration of airway lumina by granulation tissue of immature fibroblastic plugs (Masson bodies).
  4. CF - T - as a complication of repeated episodes of pulmonary infection
  5. Methotrexate is a treatment for BO - F - corticosteroids may stop progression.
  6. Lung transplant Lung, heart-lung, and BM particularly susceptable - T - chronic rejection: lung Tx, heart-lung Tx (30-50%). Chronic graft-vs-host disease, bone marrow transplant. Inflammation of bronchioles leading to (sometimes reversible) obstruction of bronchiolar lumen. Path of BO - submucosal and peribronchiolar fibrosis = irreversible fibrosis of small airway walls with narrowing / obliteration of airway lumina (respiratory bronchiole, alveolar duct, alveoli) by granulation tissue of immature fibroblastic plugs (Masson bodies) Path of BOOP / COP - granulation tissue polyps filling the lumina of alveolar ducts and respiratory bronchioles (bronchiolitis obliterans) + variable degree of infiltration of interstitium and alveoli with macrophages (organizing pneumonia). Bronchiolotiis obliterans component not present in up to 1/3rd.
91
Q
  1. CF, which is false (T/F)
  2. Fibrosing colonopathy usually ascending colon
  3. H. influenza is the most common isolated pathogen
  4. Pansinusitis
  5. Most males are infertile
  6. Survival <30y
A

false 2 and 5
45.CF, which is false (T/F) (TW)

  1. Fibrosing colonopathy usually ascending colon - T - complication of excessively high doses of pancreatic enzymes: mouth ulcerations, perianal irritation, allergic reactions, fibrosing colonopathy. Colonic strictures usually involve the ascending colon, but can be present throughout.
  2. H. influenza is the most common isolated pathogen - F - pseudomonas aeruginosa is the most common respiratory pathogen in patients with CF. Staph aureus and H influenzae are common apthogens during early childhood, but pseudomonas aeruginosa is ultimately isolated from the respiratory secretions of most patients. Data collected in a 1998 multicenter study of CF patients showed that P. aeruginosa, S. aureus, and H. influenzae could be cultured from the sputum or respiratory tract secretions of 61, 47, and 16% of tested CF patients, respectively
  3. Pansinusitis - T
  4. Most males are infertile - T - more than 95% of men with CF are infertile because of defects in sperm transport, although spermatogenesis is not affected. Most of these men have incompletely developed Wolfian structures, most commonly absent vas deferens. Indicence of female infertility may be as high as 20%.
  5. Survival <30y - F – current survival calculations averages ~40y. CF- mutation of CFTR gene (chromosome 7 q 31.2)- cystic fibrosis transmembrane regulator - skin: too much NaCl -> salt test (salty sweat)- other then skin: too little NaCl -> secretion that dries out -> thick mutation
92
Q

106.V/Q - which is the least common cause of unilateral lung perfusion

  1. Pulmonary embolism
  2. Bronchogenic carcinoma
  3. Congenital heart disease
  4. Unilateral pleural or parenchymal disease
A

3.Congenital heart disease - F - 15%

V/Q - which is the least common cause of unilateral lung perfusion: (TW)

  1. Pulmonary embolism - T - 23% - note that althought Dahnert has these as equal %, the CME 03.37 says bronchogenic carcinoma > PE.
  2. Bronchogenic carcinoma - T - 23%
  3. Congenital heart disease - F - 15%
  4. Unilateral pleural or parenchymal disease - T - 23% Incidence of unilateral lung perfusion 2%. Other causes - airway disease (bronchial adenoma, aspirated FB); arterial disease (Swyer-James, congenital pulmonary artery hypoplasia; shunt procedure eg Blalock Taussig); Absent lung (pneumonectomy, unilateral pulmonary agenesis). (Dahnert)
93
Q
  1. Klebsiella, which is false:
  2. Affects the lower lobes
  3. Pericarditis
  4. Pleural effusion
A
  1. Affects the lower lobes - F – often produes a lobar pneumonia (CAP) that is similar radiologically to that due to sreptococcus. There is a prediliction for involvement of the posterior segment of the RUL and bulging of fissue - but these are less common with nosocomial klebsiella (UTD). Posterior portion of upper lobe / superior portion of lower lobe (D).
  2. Klebsiella, which is false: (TW)
  3. Affects the lower lobes - F – often produes a lobar pneumonia (CAP) that is similar radiologically to that due to sreptococcus. There is a prediliction for involvement of the posterior segment of the RUL and bulging of fissue - but these are less common with nosocomial klebsiella (UTD). Posterior portion of upper lobe / superior portion of lower lobe (D).
  4. Pericarditis - T - klebsiella is a cause of purulent pericarditis. In the antibiotic era staph aureus is most common cause (UTD).
  5. Pleural effusion - T – one of the most common causes of empyema (D).
  6. Bulging fissure - T - due to large amounts of inflammatory exudate. Characteristic, but unusual finding. Purulent pericarditis ddx- staph- strep- klebseilla- H. influenza
94
Q
  1. Amyloid, which is false:
  2. Haemoptysis
  3. Ossification
  4. Nodules
  5. Bronchopleural fistula
A
  1. Bronchopleural fistula - F
  2. Amyloid, which is false: (TW)
  3. Haemoptysis - T – tracheobronchial type (D)
  4. Ossification - T – patients wth localised nodular amyloidosis (localised AL) are usually asymptomatic. Single or multiple lung nodules or masses, stippled or dense calcification in up to 50% cases. Diffuse parenchymal type may also have calcification, and calcification is also common in tracheobronchial amyloidosis (D and TI).
  5. Nodules - T – nodular parenchymal form
  6. Bronchopleural fistula - F
95
Q
  1. ARDS, which is false:
  2. Opacification anterior lungs
  3. CXR precedes clinical evidence of respiratory failure
  4. Effusions rare
  5. Thrombosis recognised Cx
  6. 10% residual opacities
A

false 1, 2

  1. ARDS, which is false: (TW)
  2. Opacification anterior lungs - ?F - bilateral patchy areas of air-space consolidation; these tend to have a more peripheral distribution than those seen in patients with hydrostatic oedema. They increase with time and eventually become confluent. (GL These tend to occur in the dependent aspect of lung)
  3. CXR precedes clinical evidence of respiratory failure - F – radiographs are typically normal for the first 12-24hrs after the acute injury, despite the presence of dyspnoea. This latent period is suggestive of ARDS.
  4. Effusions rare - T - Unilateral or bilateral plerual effusions may be seen, but they are typically small. No increase in pulmonary capillary pressure. Normal pulmonary artery wedge pressure and absence of elevated left atrial pressure (TI). The absence of pulmonary venous congestion… cardiomegaly and pleural effusions favors the diagnosis of ARDS (UTD).
  5. Thrombosis recognised Cx - T – pulmonary HTN can occur secondary to vasoconstriction, thrombosis, perivascular oedema and inflammation. Potential Cx - barotrauma (mechanical ventilation), nosocomial pneumonia, and multiple organ failure. Other indirect Cx include DVT, GI bleeding, malnutrition, and catheter-related infections.
  6. 10% residual opacities - ?? not sure of % - progression of consolidation after 1 week may indicated superimposed pneumonia. Over time, consolidation clears in patients with ARDS, but ground-glass opacity and findings of lung fibrosis may persist.
96
Q
  1. SLE, pulmonary changes, which is false:
  2. Pleurisy most common presenting symptom
  3. More common than with rheumatoid arthritis
  4. Pulmonary hypertension is common
  5. The arthritis is usually mild
A

***LJS opinion - pulmonary changes: diffuse alveolar haemorrhage due to vasculitis is the most common pulmonary parenchymal abnormality in SLE, followed by vanishing lung syndrome. If the question genuinely asks about pulmonary parenchymal changes then this is LESS common in SLE than RA (40% have ILD, though can be asymptomatic, versus 8% SLE pt). If they include pleural disease, then SLE more common than RA

*LW:
Agree with LJS re above statement, that pulmonary, in the sense of parenchymal changes, are LESS common than Rheumatoid arthritis. (https://www.ncbi.nlm.nih.gov/pmc/articles/PMC5964428/)

Pulmonary changes of SLE: DAD, diffuse alveolar haemorrhage, and pulmonary oedema.
Serositis is common in SLE, with pleural or pericardial effusions present in isolation without or with pulmonary disease.
Vanishing lung syndrome is a rare complication of SLE.

Pulmonary hypertension is considered UNCOMMON in SLE.

Thus both options 2 and 3 are incorrect.

Previous answer

  1. Pulmonary hypertension is common - F - pulmonary HTN is rare in SLE. It may be secondary to lung disease or pulmonary emboli or may be similar to primary pulmonary hypertension.
  2. SLE, pulmonary changes, which is false: (TW)
  3. Pleurisy most common presenting symptom - T - Pleurisy, coughing, and/or dyspnoea are often the first clues to either lung involvement or SLE itself. Occasionally abnormal RFTs and/or abnormal CXR may be detected in asymptomatic patients (UTD). Pleural disease is the most common abnormality present. Pleural effusion or pleural thickening occurs in 70% of cases (TI).
  4. More common than with rheumatoid arthritis - T - SLE affects respiratory system more commonly than any other connective tissue disease (D). However RA - 5-10% have radiologically detectable ILD. Pulmonary fibrosis is uncommon in SLE compared with other collagen-vascular diseases. Pleural involvment SLE > RA (TI)
  5. Pulmonary hypertension is common - F - pulmonary HTN is rare in SLE. It may be secondary to lung disease or pulmonary emboli or may be similar to primary pulmonary hypertension.
  6. The arthritis is usually mild - ?T - relative to what? Plus this is a pulmonary question. Arthritis and arthralgias have been noted in up to 95% of patients with SLE. Symmetrical and polyarticular. Generally considered nondeformting arthritis. Unlike RA erosions in the hand are rarely noted on radiographs. Small % (unlike RA) can have flexion deformities, soft tissue laxity, and swan neck deformities (deformities are reducible) (UTD). “nonerosive arthritis of hands (characteristic) without deformity” (D).
97
Q
  1. COP, which is true:
  2. 90% post viral
  3. Scattered consolidation on CXR
  4. Peripheral honeycombing
  5. Swyer James
  6. Distal air trapping
A
  1. Scattered consolidation on CXR - T - usually shows unilateral or bilateral patchy consolidations that resemble pneumonic infiltrates.
  2. BOOP, which is true: (TW)
  3. 90% post viral - F - aetiology: idiopathic (by definition) (DI). However, patients typically report a respiratory tract infection preceding their symptoms (Radiographics), brief flulike illness with sore throat (40%) (D). Most examples of organizing pneumonia are idiopathic (ie ‘cryptogenic) (TI).
  4. Scattered consolidation on CXR - T - usually shows unilateral or bilateral patchy consolidations that resemble pneumonic infiltrates.
  5. Peripheral honeycombing - F - honeycombing is uncommon CT finding (DI)
  6. Swyer James - F - bronchiolitis obliterans not BOOP. Chronic complication of bronchiolitis: variant of postinfectious constrictive bronchiolitis with acute obliterative bronchiolitis, bronchiectasis, distal airspace destruction (D).
  7. Distal air trapping - F - bronchiolitis obliterans vs BOOP/COP. In bronchiolitis obliterans: get irreversible fibrosis of small airway walls with narrowing / obliteration of airway lumina by granulation tissue of immature fibroblastic plugs (Masson) with “mosiac perfusion” of lobular air trapping (85-100%) + patchy air trapping on expiratory scans. In BOOP/COP - like BO with variable degree of infiltration of interstitium and alveoli with macrophages (organizing pneumonia). Bronchiolitis obliterans component not present in up to 1/3 of COP (D).
98
Q

112.Anterior mediastinal mass, which is true:

  1. Thymolipoma drapes over vessels
  2. Benign tumour, ?Spread continuous along pleural cavity
  3. MRI signal characteristics do not help differentiate benign and malignant thymoma
  4. Thymoma is the most common thymic abnormality seen with myasthenia gravis
  5. Increase in size of thymus = relapse
A
  1. Thymolipoma drapes over vessels - T – note this has also been remembered as ‘compresses vessels’ which is F. Thymolipoma can arise within thymus or be connected to thymus by a pedicle. In most cases thymolipoma is unaccompanied by sumptoms and is detected incidentally on chest radiographs - often large (avg 20cm). Fatty and pliable - and drapes over heart (TI). No compression/invasion of adjacent structures (D).
  2. Anterior mediastinal mass, which is true: (TW)
  3. Thymolipoma drapes over vessels - T – note this has also been remembered as ‘compresses vessels’ which is F. Thymolipoma can arise within thymus or be connected to thymus by a pedicle. In most cases thymolipoma is unaccompanied by sumptoms and is detected incidentally on chest radiographs - often large (avg 20cm). Fatty and pliable - and drapes over heart (TI). No compression/invasion of adjacent structures (D).
  4. Benign tumour can spread continuous along pleural cavity - F - coeloemic spread of malignant thymoma. spread by contiguity along pleural reflections, extension along aorta reaching posterior mediastinum / crus of diaphragm / retroperitoneum (D)
  5. MRI signal characteristics do not help differentiate benign and malignant thymoma - F – thymic carcinoma appears higher in signal than muscle on T1WI, with increase T2WI. Thymomas typically have a low signal intensity on T1WI (iso to muscle), which increases with T2 weighting (TI).
  6. Thymoma is the most common thymic abnormality seen with myasthenia gravis - F – most common is follicular thymic hyperplasia (65%) (DI), however MG is the most common assoc disease seen with thymoma (Radiographics 2006).
  7. Increase in size of thymus = relapse - F - thymic rebound.
99
Q

113.Which is false re the thymus:

  1. Absolute largest size reached at 10-15 years
  2. Normal thymic tissue in the neck is a recognised variant
  3. Thymoma most common in adolescents
  4. A left anterior hump is a normal finding
  5. The thymus gets smaller in response to stress
A
  1. Thymoma most common in adolescents - F - thymoma is most common primary neoplasm of anterior superior mediastinum. Majority >40yo; 70% in 5th - 6th decade; less frequent with young adults, rare in children (D).
  2. Which is false re the thymus: (TW)
  3. Absolute largest size reached at 10-15 years - T - reaches maximum absolute weight at puberty. However reaches greatest weight in proportion to body weight before birth.
  4. Normal thymic tissue in the neck is a recognised variant - T - because the thymus migrates from the 3rd and 4th branchial pouches to the anterior mediastinum, ectopic thymic tissue or ectopic thymoma can occur anywhere along this pathway.
  5. Thymoma most common in adolescents - F - thymoma is most common primary neoplasm of anterior superior mediastinum. Majority >40yo; 70% in 5th - 6th decade; less frequent with young adults, rare in children (D).
  6. A left anterior hump is a normal finding - T - puberty to age 25 thymus appears triangular or bilobed. The left lobe is usually larger, being seen lateral to aortic arch; the right lobe may be inconspicuous. Typically the lateral borders of the thymus are flat or concave where they contact th epleura; uncommonly they are slightly convex (radiographics 2006). Characteristic thymic lobe hump seen in ~30% of normal thymus glands (AJR 1983).
  7. The thymus gets smaller in response to stress - T - thymic hyperplasia may be seen when a patient is recovering from some recent stress (eg chemotherapy for neoplasm, corticosteroid therapy, irradiation, or thermal burns). Under such conditions, the thymus may respond by becoming atrophic; however, it grows back to its original size after cessation of the stress. Sometimes grows larger. Thymus arises bilaterally from 3rd and 4th branchial pouches and containes elements derived from all three germinal layers. Development begins in the 6th gestational week. Migration at 8th week.
100
Q

114.Which is an atypical thoracic manifestation of NF 1:

  1. Basal honeycombing
  2. Apical cysts
  3. Short segment scoliosis
  4. Lateral Meningocele on concave side of scoliosis
  5. Superior rib notching
A
  1. Lateral Meningocele on concave side of scoliosis - F - intervertebral foramina enlargement due to lateral thoracic meningocele, and neurofibromas. Lateral thoracic meningocele - meningocele on convex side (DI).
  2. Which is an atypical thoracic manifestation of NF 1: (TW)
  3. Basal honeycombing - T - Lung disease is present in 10-20% of adult patients with neurofibromatosis. It is characterized histologically by bullae in the upper lobes and interstitial fibrosis at the lung bases. Patients typically present with dyspnoea (TI).
  4. Apical cysts - T - see ans 1.
  5. Short segment scoliosis - T - short segment angular scoliosis. NF1: sharply angled at thoracolumbar junction (DI).
  6. Lateral Meningocele on concave side of scoliosis - F - intervertebral foramina enlargement due to lateral thoracic meningocele, and neurofibromas. Lateral thoracic meningocele - meningocele on convex side (DI).
  7. Superior rib notching - T - can cause both superior and inferior (C & N).
101
Q

61.Which isn’t bronchovascular on HRCT:

  1. Lymphangitis carcinomatosis
  2. Wegner’s
  3. Sarcoid
  4. Alveolar proteinosis
  5. LIP
A
  1. Alveolar proteinosis - F - accumulation of abundant protein-rich and lipid-rich surfactant material in alveoli. HRCT - GGO and septal thickening. Crazy paving. Interestingly although septa thickened at HRCT, septal thickening is histologically uncommon.
  2. Which isn’t bronchovascular on HRCT: (TW)
  3. Lymphangitis carcinomatosis - T
  4. Wegner’s - T - nodules in peri-bronchovascular distribution / angiocentric distribution
  5. Sarcoid - T - PL nodules: sardcoid / silicosis / CWP / lymphangitis
  6. Alveolar proteinosis - F - accumulation of abundant protein-rich and lipid-rich surfactant material in alveoli. HRCT - GGO and septal thickening. Crazy paving. Interestingly although septa thickened at HRCT, septal thickening is histologically uncommon.
  7. LIP - T - 2-4mm centrilobular and subpleural nodules in lymphatic distribution (DI).
102
Q
  1. SLE, which is false:
  2. Less likely to cause lower lobe fibrosis than RA
  3. Libman Sachs nodules do not cause valve destruction
  4. Pancreatitis secondary to vasculitis
  5. 30% of antiphospholipid syndrome has SLE
A
  1. Libman Sachs nodules do not cause valve destruction - F - Libman-sacks (verrucous) endocarditis. The verrucae, most common on the aortic and mitral valves. Accumulations of immune complexes, mononuclear cells, hematoxylin bodies, and fibrin and platelet thrombi. Typically asymptomatic, however they can fragment and produce systemic emboli (UTD). Spectrum of valvular disease in SLE ranges from valve leaflet thickening to Libman-Sacks endocarditis. The latter is characterised by formation of small single or multiple, sterile, granular pink vegetations ranging from 1-4mm that may be associated with intense valvulitis and lead to valve destruction (Radiographics 2004). Intense valvulitis and fibrinoid necrosis of the valve substance (Robbins).
  2. SLE, which is false: (TW)
  3. Less likely to cause lower lobe fibrosis than RA - T - pulmonary fibrosis is less common relative to other collagen vascular diseases.
  4. Libman Sachs nodules do not cause valve destruction - F - Libman-sacks (verrucous) endocarditis. The verrucae, most common on the aortic and mitral valves. Accumulations of immune complexes, mononuclear cells, hematoxylin bodies, and fibrin and platelet thrombi. Typically asymptomatic, however they can fragment and produce systemic emboli (UTD). Spectrum of valvular disease in SLE ranges from valve leaflet thickening to Libman-Sacks endocarditis. The latter is characterised by formation of small single or multiple, sterile, granular pink vegetations ranging from 1-4mm that may be associated with intense valvulitis and lead to valve destruction (Radiographics 2004). Intense valvulitis and fibrinoid necrosis of the valve substance (Robbins).
  5. Pancreatitis secondary to vasculitis - T - GIT is often involved with SLE, but more commonly from medication side effects than from active SLE. SLE vasculitis can lead to pancreatitis, peritonitis, and colitis (UTD).
  6. 30% of antiphospholipid syndrome has SLE - T - APS is defined by 2 major components: presence in plasma of at least one type of autoantibody known as antiphospholipid antibody (aPL); occurrence of at least one clinical feature from a diverse list of potential disease manifestations. 34% of patients with aPL have SLE. About 50% of individuals with lupus anticoagulants meed the Am College of Rheum criteria for classification of SLE.
103
Q
  1. CCAM, which is false:
  2. Can be associated with other anomalies
  3. Associated with mediastinal shift
  4. There are more than 3 types of CCAM proposed
  5. Surgery is only indicated in symptomatic patients
A
  1. Surgery is only indicated in symptomatic patients - F - this is controversial. Recommendation (UTD) of surgery for asymptomatic patients with Dx of CCM on prenatal US or indicental radiographic finding. Timing of Sx is also controversial. Excision prevents Cx such as infection and rare cases of malignancy. Lesions are also easier to resect electively than after infection (Blackbook Paeds).
  2. CCAM, which is most incorrect: (TW)
  3. Can be associated with other anomalies - ?T - fetal karyotype is indicated if there are additional structural anomalies, which are present in approx 25% of cases detected prenatally (comes down to what you think is common vs uncommon)
  4. Associated with mediastinal shift - T - can cause mediastinal shift which is a poor prognostic sign.
  5. There are more than 3 types of CCAM proposed - T - according to UTD - three major types 1,2,3 were initially based upon the size of the cysts and cellular characteristics. 2 additional types have been proposed based upon the site of origin of the malformation: Type 0 and type 4 which have tracheal and alveolar / distal acinar origins respectively.
  6. Surgery is only indicated in symptomatic patients - F - this is controversial. Recommendation (UTD) of surgery for asymptomatic patients with Dx of CCM on prenatal US or indicental radiographic finding. Timing of Sx is also controversial. Excision prevents Cx such as infection and rare cases of malignancy. Lesions are also easier to resect electively than after infection (Blackbook Paeds).
104
Q
  1. With respect to ventilation / perfusion scintigraphic lung scans:
  2. Using the PIOPED criteria for classification of results, a high probablitity scan is defined as showing one or more unmatched segmental perfusion defects
  3. Classification of a lung scan as a low probability excludes the diagnosis of PE
  4. Complete unilateral loss of perfusion is more commonly seen in extrinisic compression by bronchogenic carcinoma than in massive PE
  5. Pregnancy is an absolute contraindication for performing a scintigraphic VQ scan
A
  1. Complete unilateral loss of perfusion is more commonly seen in extrinisic compression by bronchogenic carcinoma than in massive PE - T - Acutally not sure about this one - but was True in the CME 03.37. Dahnert says 2% incidence of unilateral lung perfusion of which PE 23%, Bronchogenic carcinoma 23%. A paper from1987 (Radiographics, Cho et al) said PE is one of the least common causes, which was in agreement with another series.
  2. With respect to ventilation / perfusion scintigraphic lung scans: (TW)
  3. Using the PIOPED criteria for classification of results, a high probablitity scan is defined as showing one or more unmatched segmental perfusion defects - F - 2 or more large (segmental) perfusion defects without match
  4. Classification of a lung scan as a low probability excludes the diagnosis of PE - F - Low probability of PE accounted for 34% cases, and 16% of these were positive for PE on angiogram. Even normal VQ (14% cases) had angiogram positive findings in 9%.
  5. Complete unilateral loss of perfusion is more commonly seen in extrinisic compression by bronchogenic carcinoma than in massive PE - T - Acutally not sure about this one - but was True in the CME 03.37. Dahnert says 2% incidence of unilateral lung perfusion of which PE 23%, Bronchogenic carcinoma 23%. A paper from1987 (Radiographics, Cho et al) said PE is one of the least common causes, which was in agreement with another series.
  6. Pregnancy is an absolute contraindication for performing a scintigraphic VQ scan - F - pregnancy has 5x increase risk of VTE, and PE is leading preventable cause of maternal death. Mixed recommendations - it is VQ is favored by PIOPED II investigators as a means of imaging in pregnancy, has lower breast dose and lower nondiagnostic rate in pregnant population. Can reduce dose by eliminating the ventilation portion of examination in patients with normal perfusion (plus whack an IDC in to stop the bladder-fetus exposure). Of note - ther test: D-dimer can have false-positive results as pregnancy progresses, however NPV remains accurate regardless of trimester (but PE has still been found in normal D-dimers). Fetal risks from radiation doses of less than 50mGy are negligible, and doss of 100mGy result in combined increased risk of organ malformation and development of childhood cancer of only about 1%.
105
Q

119.CCAM, which is false:

  1. Associated anomalies are present in the majority of cases
  2. Associated with mediastinal shift
  3. Microcystic form has a worse prognosis than macrocystic form
  4. Associated with normal situs
  5. Majority of cases detected in fetus are associated with polyhydramnios and ascites
A
  1. Associated anomalies are present in the majority of cases - F - fetal karyotype is indicated if there are additional structural anomalies, which are present in approx 25% of cases detected prenatally.
  2. CCAM, which is false: (TW)
  3. Associated anomalies are present in the majority of cases - F - fetal karyotype is indicated if there are additional structural anomalies, which are present in approx 25% of cases detected prenatally.
  4. Associated with mediastinal shift T – can cause mediastinal shift which is a poor prognostic sign (UTD). 89% have contralateral mediastinal shift (D).
  5. Microcystic form has a worse prognosis than macrocystic form - T - for the majority of type 1 lesions, surgical excision in neonatal period is curative and prognosis is excellent. Outcome for type 4 CCAM (there are now 5 proposed types 0-4) is also quite good. Prognosis for type 2 and 3 is not as favorable. Type 0 represents a severe derangement of fetal development as is assoc with other anomalies (not compatible with survival).
  6. Associated with normal situs - T
  7. Majority of cases detected in fetus are associated with polyhydramnios and ascites - T - polyhydramnios + ascites = 2 abnormal fetal fluid accumulations = hydrops. CME says answer is true, but i think debatable. Dahnert - polyhydramnios 25-75%. Blackbook - says usually incidental finding. UTD says hydrops deveps in as many as 40% cases
106
Q

120.Regarding primary TB of the lung:

  1. 90% have consolidation
  2. Anterior segment of the upper lobe most commonly
  3. Pleural effusion in adults 30%
  4. Hilar adenopathy in children 30%
A
  1. Pleural effusion in adults 30% - T - 23-38% adults, 10% kids (D).
  2. Regarding primary TB of the lung: (TW)
  3. 90% have consolidation - F - patients with primary MTB most often show no radiologic abnormalities. If overt infection occurs, the pattern is usually one of air-space consolidation, often involving an entire lobe. Atelectasis often encountered in children with primary MTB (?related to airway compression by large nodes) (TI, DI). Note that CME 99.43 had primary lung infiltrate most commonly lower lobe as true.
  4. Anterior segment of the upper lobe most commonly - F - see ans 1. also. If consolidation occurs, right lung more common than left, although no definite zonal predominance is seen. Progressive primary TB (rare) can lead to extensive consolidation an dcavitation, either at site of initial pulmonary parenchymal focus of infection, or in the apical and posterior segments of the upper lobes. In post-primary MTB consolidation favors the apical and posterior segments of the upper lobes, and to a lesser extend the superior segment of the lower lobes (TI). Radiographics (TB review 2007) says predominance in lower and middle lobes (esp adults), and kids atelectasis most often anterior segment upper lobe, of medial segment of middle lobe.
  5. Pleural effusion in adults 30% - T - 23-38% adults, 10% kids (D).
  6. Hilar adenopathy in children 30% - F - Children 60%; Adults 5-48%. (D)
107
Q

124.Which of the following is not a cause of a cavitating lung lesion:

  1. Kelbsiella pneumonia
  2. Actinomycosis israelii
  3. Wegeners granulomatosis
  4. Small cell lung cancer
A
  1. Small cell lung cancer - F - Although cavitation may be found in tumours of any size or histological type, it is most commonly associated with SCC, and is rarely seen in small cell carcinomas (Lung Cancer, S Desai).
  2. Which of the following is least likely cause of a cavitating lung lesion: (TW)
  3. Klebsiella pneumonia - T - thick-walled with ragged inner lining. More common in upper lobes (C&N)
  4. Acitinomycosis israelii - T - anaerobic or microaerophilic organism. Forms mycelia that, in tissue, may cluster to form sulfur granules (named due to colour, as actually only contain little actual sulphur). May present as a mass, often with cavitation, simulating lung carcinoma. Chest wall invasion may also occur. Pattern particularly suggestive of A. israelii chest wall infection is wavy periosteal reaction affecting several contiguous ribs (TI).
  5. Wegeners granulomatosis - T - cavitation of nodules with thick wall and irregular shaggy inner lining (25-50%) (D).
  6. Small cell lung cancer - F - Although cavitation may be found in tumours of any size or histological type, it is most commonly associated with SCC, and is rarely seen in small cell carcinomas (Lung Cancer, S Desai). Added options 1,2,3.
108
Q

126.The following statements are true:

  1. About 80% of bronchial carcinoids are located peripherally
  2. Curvilinear calcification is virtually pathognomic of pulmonary hamartoma
  3. ARDS rarely develops in the absence of pre existing lung disease
  4. Redistribution of blood flow to the upper zones occurs in ARDS
  5. Extralobar sequestration drains via the systemic venous system
A
  1. Extralobar sequestration drains via the systemic venous system - T - typically drains to RA, IVC, or azygous system. ILS typically drains to pulmonary veins and LA, however can also have abnormal connection like ELS.
  2. The following statements are true: (TW)
  3. About 80% of bronchial carcinoids are located peripherally - F - tumor most often occurs in the central bronchi, endobronchial mass, and is locally invasive (TI). Typical carcinoid: 85% develop in main, lobar or segmental bronchi; 15% are peripheral in location. Atypical carcinoid: most develop in lung periphery (DI). Neuroendocrine differentiation of the Kulchisky cells of bronchial mucosa and resemble intestinal carcinoids (Robbins).
  4. Curvilinear calcification is virtually pathognomic of pulmonary hamartoma - F - conglomerate or ‘popcorn’ calcification is characteristic of hamartomas and is rarely seen with other lesions; it occurs because of calcification of nodules of cartilage (TI). Fat density in 50% (diagnostic) (D).
  5. ARDS rarely develops in the absence of pre existing lung disease - F - ARDS can be related to a variety of pathologic processes, incl
  6. infection,
  7. inhalation or aspiration of toxic or irritating substances,
  8. trauma with lung or extrathoracic injury,
  9. hemodynamic abnormalities (eg shock, high-altitude),
  10. hematologic disorders (DIC),
  11. embolic disease (fat, amniotic),
  12. Drugs,
  13. metabolic (pancreatitis, ketoacidosis),
  14. neurologic disease (head injury, stroke) (TI).
  15. Redistribution of blood flow to the upper zones occurs in ARDS - F - part of the specific criteria for ARDS is normal pulmonary artery wedge pressure and absence of elevated left atrial pressure (other criteria are: acute onset, hypoxemia despite high FiO2, characteristic bilateral radiographic abnormalities) (TI).
  16. Extralobar sequestration drains via the systemic venous system - T - typically drains to RA, IVC, or azygous system. ILS typically drains to pulmonary veins and LA, however can also have abnormal connection like ELS.
109
Q

127.Allergic bronchopulmonary aspergillosis (T/F):

  1. Hilar lymph node enlargement is a recognised feature
  2. Mycetoma (aspergilloma) is seen in approximately 1/3rd of patients
  3. Moderate of large pleural effusions are seen in >1/4 of patients
  4. The bronchiectasis is characteristically central in location
  5. The bronchiectasis is characteristically in the upper lobes
A

true 4 and 5

  1. Allergic bronchopulmonary aspergillosis (T/F): (TW)
  2. Hilar lymph node enlargement is a recognised feature - F - CME 02.84
  3. Mycetoma (aspergilloma) is seen in approximately 1/3rd of patients - F - unusual are aspergilloma in cavity (7%) (D).
  4. Moderate or large pleural effusions are seen in >1/4 of patients - F - CME 02.84
  5. The bronchiectasis is characteristically central in location - T - bronchiectasis is common visible with acute or recurrent disease. The abnormal bronchi often are lobar or segmental and, thus, central in location. Presence of a dilated thick-walled bronchus sometims is termed bronchocele, whereas a bronchocele containing a mucous plug may be referred to as a mucocele. These tend to ahve an upper lobe predominance in A. Radiographic findings in ABPA (eg, central bronchiectasis with an upper lobe predominance, mucus plugging, consolidation, large lung voluems) may mimic those of CF (TI).
  6. The bronchiectasis is characteristically in the upper lobes - T - see ans 4. ABPA reflects a hyperesnsitivity reactio nto Apergillus organisms and characteristically is associated with eosinophilia, symptoms of asthma such as wheezing, and findings of ‘centra’ or ‘proximal’ bronchiectasis, usually with mucoid imaction, atelectasis, and consolidation similar to that seen in patients with eosinophilic pneumonia. ABPA results from both type I (IgE) and type III (IgG) immunologic reactions (TI).
110
Q

129.The following statements regarding thoracic neoplasms in AIDS patients are true:

  1. Positive gallium scan distinguishes lymphoma from Kaposi sarcoma
  2. Kaposi sarcoma is typically peripheral in distribution
  3. Lymph nodes involved with Kaposi’s sarcoma don’t enhance strongly post IV contrast
  4. Pleural effusions occur in about 5% of patients with pulmonary Kaposi sarcoma
  5. There is a strong association of AIDS with Hodgkin’s lymphoma
A
  1. Positive gallium scan distinguishes lymphoma from Kaposi sarcoma T - thallium is positive in both, but KS is not Ga-avid. 70.The following statements regarding thoracic neoplasms in AIDS patients are true: (GC)
  2. Positive gallium scan distinguishes lymphoma from Kaposi sarcoma T - thallium is positive in both, but KS is not Ga-avid.
  3. Kaposi sarcoma is typically peripheral in distribution F - Mid to lower zones, peribronchial/perivascular axial interstitium.
  4. Lymph nodes involved with Kaposi’s sarcoma don’t enhance strongly post IV contrast F - strongly enhancing nodes are seen.
  5. Pleural effusions occur in about 5% of patients with pulmonary Kaposi sarcoma F - occur in 1/3 to 2/3 of patients. The presence of pleural collections has been associated with shortened survival times.
  6. There is a strong association of AIDS with Hodgkin’s lymphoma F - generally high grade lymphomas of B-cell origin; thought to be related to immunosuppression, although cause remains unclear. Incidence of NHL in AIDS patients was 60-200 times higher than in a matched HIV-negative population prior to the introduction of HAART, and the RR was even greater for primary cerebral lymphoma. Hodgkin’s disease is occasionally seen. [Kaposi Sarcoma RG 2006; AIDS-related NHL, QJM 2006; Dahnert
111
Q

130.The following statements are true of diseases affecting the anterior mediastinum (T/F)

  1. Invasive thymomas do not calcify
  2. Large thymolipomas invade surrouding structures
  3. Thymic cysts are found in patients with Hodgkin’s disease
  4. Thymic hyperplasia is a feature of recovery from a childhood illness
  5. Lymphoma confined to the anterior mediatsinum is most likely to be non Hodgkin’s lymphoma
A
  1. Thymic hyperplasia is a feature of recovery from a childhood illness - T - thymus shrinks during illness, and rebounds after.
  2. The following statements are true of diseases affecting the anterior mediastinum (T/F) (TW)
  3. Invasive thymomas do not calcify - F - calcification may occur in the capsule or in within the tumor (both invasive and noninvasive) (TI). Calcification is usually thin, linear and peripheral (DI).
  4. Large thymolipomas invade surrouding structures - F - rare, benign, well-encapsulated tumor consisting primarily of fat, but containing variable amounts of thymic tissue. Usually asymptomatic. Is of ten large averaging nearly 20cm in diameter at Dx. Tends to drape over heart. Does not invade (TI, D).
  5. Thymic cysts are seen in Hodgkins disease F - Thymic cysts are uncommon - can be eithe rcongenital or acquired. Acquired TCs reported following radiotherapy and in association with thymic tumors, and following thoracotomy. Thymic lymphoma usually results in homogeneous thymic enlargement, however lobulation or nodular appearance is seen in some patients, and cystic areas of necrosis are visible in 20% on CT of adults.
  6. Thymic hyperplasia is a feature of recovery from a childhood illness - T - thymus shrinks during illness, and rebounds after.
  7. Lymphoma confined to the anterior mediatsinum is most likely to be non Hodgkin’s lymphoma - F - Hodgkin’s disease has a predilection for involvement of teh thymus in conjunction with involvement of mediastinal lymph nodes. NHL much less commonly involves the thymus.
112
Q
  1. The following associations regarding drug induced lung disease are recognized, which is false:
  2. Methotrexate and hilar adenopathy
  3. Amiodarone and predominantly perihilar opacities
  4. Cyclosporin and a solitary pulmonary mass
  5. Ampicillin and eosinophilic pneumonia
  6. Cocaine abuse and intra alveolar haemorrhage
A
  1. Amiodarone and predominantly perihilar opacities - F - pulmonary toxicity in 5% patients. Varied radiographic patterns - focal or diffuse areas of consolidation, reticular opacities, and les comonly ill-defined nodules or masses. Peripheral reticular opacities. NSIP (most common) (TI, D).
  2. The following associations regarding drug induced lung disease are recognised: (TW)
  3. Methotrexate and hilar adenopathy - T - MTX causes pulmonary reactions in 1-5% of patients. An acute hypersensitivity pneumonitis is most common. Other reactions in clude pleuritis, hilar adenopathy, and nodules.
  4. Amiodarone and predominantly perihilar opacities - F - pulmonary toxicity in 5% patients. Varied radiographic patterns - focal or diffuse areas of consolidation, reticular opacities, and les comonly ill-defined nodules or masses. Peripheral reticular opacities. NSIP (most common) (TI, D).
  5. Cyclosporin and a solitary pulmonary mass - T
  6. Ampicillin and eosinophilic pneumonia - T - penicillamine, sulfasalazine, nitrofurantoin, NSAIDs, paraaminosalicylic acid (D). Ampicillin, anticonvulsants, ACE, b-blockers, HCT, MTX, amiodarone,
  7. Cocaine abuse and intra alveolar haemorrhage - T - hemoptysis has been reported in 6-26% of crack users and is considered to be either (a) secondary to rupture of bronchial or tracheal submucosal blood vessels, or (b) parenchymal in origin, arising from injury to the alveolar-capillary membrane (Radiographics 2007) CME 03.01 Amiodarone lung:Plain radiograph Appearances on chest radiography are non-specific, typically consisting of:peripheral areas of consolidatio nupper lobe predominance underlying interstitial disease CTAs with other pulmonary diseases with an interstitial component, HRCT is the modality of choice. Changes are usually bilateral, asymmetrical and particularly prominent in the lung bases 6. Findings include:areas of consolidation peripheral often hyperdense 1,6 c.f. muscle (on account of the iodine)patchy ground-glass opacities co-existing interstitial disease reticulonodular opacities In addition, the liver (80% of cases) and sometimes the heart (20%) are high density 6. However, high hepatic and splenic attenuation is also seen in patients exposed to amiodarone in the absence of drug toxicity.
113
Q

132.Pleural effusions are seen in <10% of patients with PCP

A

73.Pleural effusions are seen in <10% of patients with PCP 1.T – suggests another cause; B&H says <5%

114
Q
  1. Which is true regarding childhood primary pulmonary TB:
  2. Pleural effusions are rare
  3. Bilateral hilar lymphadenopathy occurs in the majority
  4. Calcification accompanying healing is not typical
  5. Presence of atelectasis is associated with bronchiectasis
  6. The lung focus is typically apical
A

1.Pleural effusions are rare - T - ?rare but best answer? - D says 10% in kids, TI says patients with primary MTB most often show no radiologic abnormalities. Atelectasis is often encountered in children with pMTB and may be relate dto aireway compression by enlarged lymph nodes. Pleural effusions may occur in patient with pMTB infection . Often when TB is discovered as the cause for pleural effusion, no parenchymal focus of disease is radiographically evident; this pattern is considered characteristic of pMTB pleural infection. Such effusions are usually small and unilateral.

115
Q

136.Fat emboli, which is least correct:

  1. Slowly resolve
  2. Perihilar
  3. Extent of lung opacification generally reflects severity of disease
  4. Can have a non-traumatic aetiology
A
  1. Perihilar - F - radiography: nonspecific, diffuse parenchymal opacities without zonal predilection. In mild fat embolism - small (3-5mm) nodular opacities tend to be located predominantly in the centrilobular and subpleural regions.
  2. Fat emboli: (false) (TW)
  3. Slowly resolve - T - resolution of CT abnormalities occurs in about 2 weeks, with a range from 1-3 weeks.
  4. Perihilar - F - radiography: nonspecific, diffuse parenchymal opacities without zonal predilection. In mild fat embolism - small (3-5mm) nodular opacities tend to be located predominantly in the centrilobular and subpleural regions.
  5. Extent of lung opacification generally reflects severity of disease - T
  6. Can have a non-traumatic aetiology - T - fatty liver; prolonged corticosteroid therapy; acute pancreatitis; osteomyelitis; conditions causing bone infarcts (eg sickle cell disease). (DI)
116
Q
  1. Positive Gallium scan of the lung is not seen in:
  2. Lymphoma
  3. Sarcoid
  4. Cardiac amyloid
  5. Wilm’s tumour
  6. PCP
A

false 4.Wilm’s tumour

  1. Positive Gallium scan of the lung is seen in: (GC)
  2. Lymphoma T - helps determine residual disease versus inactive fibrosis. Uptake in Hodgkin’s disease (nodular sclerosing and mixed cellularity) - persistent uptake during and after treatment is a poor prognostic sign and suggests a high risk for relapse. Also may have uptake in NHL (intermediate and high grade).
  3. Sarcoid T - pulmonary uptake correlates with disease activity and reponse to therapy, reported to be up to 97% sensitive for detection of active sarcoidosis when considering both pulmonary and extrapulmonary sites. A grading system (0-4) may be used to try to quantitate - 0 is normal, 1 equivocal, 2 - 4 represent abnormal activity that is less than/equal to/greater than hepatic activity. [see diagram in B&H, pg 1442]
  4. Cardiac amyloid T - intense and diffuse uptake
  5. Wilm’s tumour F
  6. PCP T - intense diffuse uptake is indicative of PCP, particularly useful in HIV patients.
117
Q

139.Regarding the thymus

  1. Thymolipomas compress mediastinal structures
  2. Benign thymomas spread along pleural reflections
  3. Thymic hyperplasia is the most common cause of a mediastinal mass in an adolescent patient
  4. The MRI signal on T2 weighted images is the most reliable way to distinguish a normal thymus from thymic pathology
  5. An increase in thymic size post successful treatment of lymphoma is an indication of disease recurrence
A

*AJL Lymphoma is the most common cause of a mediastinal mass in paediatrics, I’m unsure if this can be extrapolated to adolescents.

  • *LW: I think it can be extrapolated, based on quote from UTD:
  • –> Up to 75 percent of children with HL have a mediastinal mass on chest radiograph at the time of presentation. They are more common among children older than 12 years of age, in whom approximately 30 percent have masses greater than one-third the diameter of the intrathoracic cavity.

*AL continued….
In any case, thymic hyperplasia is not super common, possibly benign enlargement of the thymus could be considered but not hyperplasia.
Thymolipomas can compress structures when they are large (and therefore become symptomatic) but early on they do not. I’m unsure of the correct answer.

Previous answer:
3.Thymic hyperplasia is the most common cause of a mediastinal mass in an adolescent patient T

  1. Regarding the thymus (TW)
  2. Thymolipomas compress mediastinal structures - F - Thymolipoma can arise within thymus or be connected to thymus by a pedicle. In most cases thymolipoma is unaccompanied by sumptoms and is detected incidentally on chest radiographs - often large (avg 20cm). Fatty and pliable - and drapes over heart (TI). No compression/invasion of adjacent structures (D).
  3. Benign thymomas spread along pleural reflections - F - coeloemic spread of malignant/invasive thymoma. spread by contiguity along pleural reflections, extension along aorta reaching posterior mediastinum / crus of diaphragm / retroperitoneum (D)
  4. Thymic hyperplasia is the most common cause of a mediastinal mass in an adolescent patient T
  5. The MRI signal on T2 weighted images is the most reliable way to distinguish a normal thymus from thymic pathology - F- thymic carcinoma appears higher in signal than muscle on T1WI, with increase T2WI. Thymomas typically have a low signal intensity on T1WI (iso to muscle), which increases with T2 weighting (TI).
  6. An increase in thymic size post successful treatment of lymphoma is an indication of disease recurrence - F - thymic rebound. Thymus shrinks during illness / Rx, and enlarges after illness - can get bigger than original size.
118
Q

140.Regarding mesothelioma:

  1. Peritoneal mesothelioma is not associated with asbestos exposure
  2. Peritoneal mesothelioma shows calcification in 90%
  3. Peritoneal mesothelioma occurs in middle aged men
  4. Pleural mesothelioma only arises from the parietal layer of pleura
  5. One third of mesothelioma occurs in the peritoneum
A

**LW:
StatDx states: 20-30% of malignant mesotheliomas arise in peritoneum, is related to asbestos exposure, Only 20% of pleural mesotheliomas have calcified plaques, and Parietal > visceral pleura involved, so option 5 CORRECT.
UTD states vast majority of MPM occur in patients age 60 years and older, typically presenting decades after an exposure to asbestos with gradually worsening, nonspecific pulmonary symptoms. so option 3 FALSE.

  • AJL - I would favour mesothelioma occuring in middle aged men (20-40 years post exposure). RP says peritoneal mesothelioma occurs 10% of the time. Robbins does not give a number.
    5. One third of mesothelioma occurs in the peritoneum - T - Dahnert says pleural 67%, peritoneum 30-40%, pericardium 2.5%, processus vaginalis 0.5%. Note that UTD says in USA, peritoneal mesothelioma accounts for only 10-15%.
    81. Regarding mesothelioma: (TW)
    1. Peritoneal mesothelioma is not associated with asbestos exposure - F - there is a strong relationship between asbestos exposure and develoment of mesothelioma at any location.
    2. Peritoneal mesothelioma shows calcification in 90% - F - calcifications within diffuse metastatic peritoneal mesothelioma are considered rare. Calcified plaques are seen less often than with pleural mesothelioma, however calcified pleural plaques and other signs assoc with asbestos exposure may be present in the chest in up to 50% of patients (UTD).
    3. Peritoneal mesothelioma occurs in middle aged men - F - median age at presentation is 63 (just over middle age), but still younger than that of the average patient with pleural mesothelioma.
    4. Pleural mesothelioma only arises from the parietal layer of pleura - F - can arise from either (Robbins)
    5. One third of mesothelioma occurs in the peritoneum - T - Dahnert says pleural 67%, peritoneum 30-40%, pericardium 2.5%, processus vaginalis 0.5%. Note that UTD says in USA, peritoneal mesothelioma accounts for only 10-15%. Cystic mesothelioma is a rare benign neoplasm without metastatic potential but has tendency for local recurrence. It is NOT associated with asbestos exposure.
119
Q

141.Regarding congenital hernia of the diaphragm T/F:

  1. Congenital abnormalities are more common with Bochdalek hernias
  2. Are associated with mediastinal shift on antenatal ultrasound
  3. The lung is the same echogenicity as the liver on antenatal ultrasound
  4. The commonest association of CDH is with anencephaly
  5. The blood supply to bronchial atresia is from the descending thoracic aorta
A

**LJS opinion: 5 is not true. Bronchial atresia is an isolated airway abnormality with normal arterial and venous systems. Yes there is one case report (the one they quote) of a mixed lesion of combined sequestration with bronchial atresia, which has arterial supply, however this is the exception rather than the rule

*LW:
overall this is ridiculous level of detail….

Difficult re: option 5:
Prresumed associated pathogenesis - primitive PA already present as a major branch and grows towats gorwing lung bud in utero. It then gives off branches to accompany each airway branch. Thus if there is an abnormality in the airway branch / atresia, no stimulus for PA to grow into that segment, hence vascular abnormalities have been documented with bronchial atresia.

Previous answer:
true 2, 3, 5

  1. Regarding congenital hernia of the diaphragm, T/F: (GC)
  2. Congenital abnormalities are more common with Bochdalek hernia F - Morgagni often associated with congenital heart disease, bowel malrotation, chromosomal abnormality (Downs, Turners), mental retardation, pericardial deficiency. Bochdaleck associated with herniated organs, more common on left (80%).
  3. Are associated with mediastinal shift on antenatal ultrasound T - mixed echogenic mass seen in hemithorax, displacement of cardiac structures, absence of stomach bubble in normal location.
  4. The lung is the same echogenicity as the liver on antenatal ultrasound T - right-sided CDH are usually more difficult to detect. Typically, only the liver herniates into the chest, its echogenicity being similar to that of the mediastinum and nonaerated lungs. If anechoic structures such as the heaptic vascular structures (the hepatic and portal veins), the gallbladder, and fluid-filled bowel also enter the chest, detection is easier. [Ultrasound Requisites 2nd ed]
  5. The commonest association of CDH is with anencephaly F - Pulmonary hypoplasia due to lung compression. High incidence of cardiac anomalies (up to 25%). Chromosomal abnormalities in up to 30% (eg. trisomies 13, 18, 21, Turner).
  6. The blood supply to bronchial atresia is from the descending thoracic aorta T - In the early stages of lung development, the tips of the dividing bronchial buds are supplied by a systemic capillary plexus derived from the primitive aorta, which regresses with further development of lung and pulmonary artery. The timing and severity of the insult are important in determining the morphology of the lesion. The cause of bronchial atresia is controversial. Because the bronchial tree distal to atresia branches normally, it has been postulated that the insult may have occurred after completion of the process of airway development at about 16 wks. However, the coexistence of other congenital defects that occur early in embryological development supports the hypothesis that an insult occurs during the stage of lung budding resulting in a localized abnormality followed by normal development of the distal bronchial tree. The persistent systemic arterial supply can be explained by the arrest of pulmonary arterial growth. In the absence of pulmonary blood supply, the systemic vessels provide the only support for the developing lung. [An unusual case of systemic arterial supply to the lung with bronchial atresia, AJR 2005]
120
Q
  1. Hilar lymphadenopathy is seen in:
  2. Cyclophosphamide
  3. Adriomycin
  4. Bleomycin
  5. Methotrexate
  6. Phenytoin
A
  1. Methotrexate - T - transient hilar adenopathy
  2. Phenytoin - T Drugs that cause hilar / mediastinal lymphadenopathy (www.pneumotox.com)Bleomycin, phenytoin, methotrexate, cyclosporine, carbamazepine, cotrimoxazole, indomethacin, b-interferon, minocycline, penicillins, etc.
  • AJL according to the above list, also:
    3. Bleomycin

Perhaps the question should be which does not cause hilar lymphadenopathy
Answer - Adriomycin and cyclophosphamide (Unless they’re included in the etc. component)

121
Q

143.Pulmonary Emboli:

  1. Septic emboli most commonly arise from the pelvic veins
  2. The defect on chest x-rays is often larger than the V/Q defect
  3. Matched defects on V/Q scans exclude PE
  4. PE is associated with pulmonary infarction in 10%
  5. The presence of a pleural effusion indicates pulmonary infarction
A
  1. PE is associated with pulmonary infarction in 10% - T - embolism with infarction 10-60%, without infarction 90% (D). UTD and path notes say 10%.
  2. Pulmonary Emboli: (TW)
  3. Septic emboli most commonly arise from the pelvic veins - F - infected venous catheter / pacemaker wires, AV shunts for hemodialysis, drug abuse producing septic thrombophlebitis, pelvic thrombophlebitis, peritonsiliar abscess, osteomyelitis, tricuspid valve endocarditis (most common cause in IVDU).
  4. The defect on chest x-rays is often larger than the V/Q defect - F - can often have normal CXR. If high probability, perfusion defects are substantially larger than CXR abnormalities.
  5. Matched defects on V/Q scans exclude PE - F - perfusion defect with lung disease (ie absent ventilation) can reflect lung infarction / pneumonia / atelectasis (correlation with CXR required) and even then, this is an inderminate result. Even if ‘normal’ study - still have PE in 14%. Indeterminate VQ lung scans - 89% have Q defect&raquo_space; CXR consolidation.
  6. PE is associated with pulmonary infarction in 10% - T - embolism with infarction 10-60%, without infarction 90% (D). UTD and path notes say 10%.
  7. The presence of a pleural effusion indicates pulmonary infarction - F
122
Q

144.There are associations between the following: which is false

  1. Streptococcus and round pneumonia
  2. Legionella and pneumatocoeles
  3. Anaerobic organisms and cavitation
  4. Mycoplasma and lymphadenopathy
A
  1. Legionella and pneumatocoeles - F - Legionella pneumonia often causes peripheral focal consolidation that rapidly progresses to involve an entire lobe or perhaps several lobes on the side of initial presentation. Cavitation is uncommon in immunocompromised (TI). Pneumatoceles assoc with infection: pneumococci, E coli, Klebsiella, Staph (in childhood) (D).
  2. There are associations between the following: (TW)
  3. Streptococcus and round pneumonia - T - typically occurs with streptococcal pneumoniae infection. In children, collateral pathways of air circulation are not well developed until approx 8yo (Channels of Lambert, Pores of Kohn). Lack of well developed collateral circulation is thought to hinder spread of bacterial infection and predispose to ‘round’ appearance. After 8yo, if round mass seen on CXR - high suspicion for other pathology.
  4. Legionella and pneumatocoeles - F - Legionella pneumonia often causes peripheral focal consolidation that rapidly progresses to involve an entire lobe or perhaps several lobes on the side of initial presentation. Cavitation is uncommon in immunocompromised (TI). Pneumatoceles assoc with infection: pneumococci, E coli, Klebsiella, Staph (in childhood) (D).
  5. Anaerobic organisms and cavitation - T - cavitation pneumonia: staph, HI, S pneumoniae, other gram-neg organisms. Cavitating opportunistic infections: fungal,; septic emboli (anaerobic organisms); staph abscess; TB (D). Anaerobic bacteris - normally found in oral cavity. Aspiration - bronchopneumonia pattern, often assoc with tissue ncerosis which produces the radiographic evidence of cavitation (TI).
  6. Mycoplasma and lymphadenopathy - T - hilar adenopathy 7-22% (D). Mycoplasmas are the smallest free-living culturable organisms. They share some similarities with
123
Q

145.Klebsiella: which is false

  1. Affects the lower lobes most frequently
  2. Show frequent and early cavitation
  3. Are associated with effusions
  4. Cause volume expansion
  5. Causes interstitial opacity
A
  1. Affects the lower lobes most frequently - F - propensity for posterior portion of upper lobe / superior portion of lower lobe (D)
  2. Klebsiella: (TW)
  3. Affects the lower lobes most frequently - F - propensity for posterior portion of upper lobe / superior portion of lower lobe (D)
  4. Show frequent and early cavitation - T - uni- / multilocular cavities (50%) appearing within 4 days (D).
  5. Are associated with effusions - T - pulmonary abscess formation is common, as is pleural effusion and empyema (TI). One of the most common causes of empyema (D).
  6. Cause volume expansion - T - dense lobar consolidation. Bulbing of fissure (large amounts of inflammatory exudate) (D).
  7. Causes interstitial opacity - F - dense lobar consolidation (D). Presnts iwth an air-space pattern, nonsegmental homogeneous consolidation resembling pneumococcal pneumonia. Air bronchograms common seen (TI).
124
Q
  1. Regarding Bronchopulmonary Sequestration, which is least correct:
  2. 80% are intra lobar
  3. Intralobar bronchopulmonary sequestration has pulmonary venous drainage
  4. Extralobar sequestration can have air bronchograms
  5. Extralobar are most common in the left hemithorax
  6. Intralobar present in the neonate
A
  1. Intralobar present in the neonate - F - ELS more commonly Dx in newborns or infants. ILS usually present in late childhood or adolescence with recurrent pulmonary infections
  2. Regarding Bronchopulmonary Sequestration, which is least correct: (TW)
  3. 80% are intra lobar - T - 75-86% (D). ILS occurs more commonly (70%) vs ELS (30%). ELS is almost always LLL. In ILS LLL (60%) > RLL (40%).
  4. Intralobar bronchopulmonary sequestration has pulmonary venous drainage- T - drainage usually to left atrium, although abnormal connections to IVC, azygous vein, or RA may occur. ELS however typically have anomalous venous drainage to RA, vena cava, or azygous system
  5. Extralobar sequestration can have air bronchograms- T - sequestration lacks normal communication with tracheobronchial tree (ILS and ELS), although ILS, albeit uncommon, may have abnormal connection. Note that both could potentially have air bronchograms as both can have connection to GIT, however generally air bronchograms are absent.
  6. Extralobar are most common in the left hemithorax - T - ELS almost always involves the left hemithorax (UTD), usually LLL.
  7. Intralobar present in the neonate - F - ELS more commonly Dx in newborns or infants. ILS usually present in late childhood or adolescence with recurrent pulmonary infections
125
Q
  1. Allergic bronchopulmonary aspergillosis (T/F)
  2. Hilar lymphnode enlargement can be a feature
  3. Mycetoma is seen in about a third
  4. Moderate-large pleural effusions seen in more than 1/4 patients
  5. Causes central bronchiectasis
  6. Upper lobe bronchiectasis is characteristic
A
  1. Causes central bronchiectasis - T - bronchiectasis is common visible with acute or recurrent disease. The abnormal bronchi often are lobar or segmental and, thus, central in location. Presence of a tilated thick-walled bronchus sometims is termed bronchocele, whereas a bronchocele containing a mucous plug may be referred to as a mucocele. These tend to ahve an upper lobe predominance in A. Radiographic findings in ABPA (eg, central bronchiectasis with an upper lobe predominance, mucus plugging, consolidation, large lung voluems) may mimic those of CF (TI).
  2. Upper lobe bronchiectasis is characteristic - T - see ans 4.
  3. Allergic bronchopulmonary aspergillosis (T/F): (TW)
  4. Hilar lymph node enlargement can be a feature - F - CME 02.84
  5. Mycetoma is seen in about a third - F - unusual are aspergilloma in cavity (7%) (D).
  6. Moderate-large pleural effusions seen in more than 1/4 of patients - F - CME 02.84
  7. Causes central bronchiectasis - T - bronchiectasis is common visible with acute or recurrent disease. The abnormal bronchi often are lobar or segmental and, thus, central in location. Presence of a tilated thick-walled bronchus sometims is termed bronchocele, whereas a bronchocele containing a mucous plug may be referred to as a mucocele. These tend to ahve an upper lobe predominance in A. Radiographic findings in ABPA (eg, central bronchiectasis with an upper lobe predominance, mucus plugging, consolidation, large lung voluems) may mimic those of CF (TI).
  8. Upper lobe bronchiectasis is characteristic - T - see ans 4.
126
Q

148.Kaposi Sarcoma in the Chest: t/f

  1. Has hypervascular lymphadenopathy
  2. Has a peripheral distribution
  3. Has effusions in less than 5%
  4. Gallium is able to differentiate this from lymphoma
A

true 1,4

  1. Kaposi Sarcoma in the Chest: (TW)
  2. Has hypervascular lymphadenopathy - T - enhancing lymph nodes can be seen
  3. Has a peripheral distribution - F - parahilar or peribronchovascular distribution.
  4. Has effusions in less than 5% - F - pleural effusion 33-67%, chylothorax is rare
  5. Gallium is able to differentiate this from lymphoma - T - gallium uptake is usually negative in KS but positive in infection and lymphoma, whereas thallium uptake is positive in KS and lymphoma. Kaposi sarcoma: Low grade mesenchymal tumor that involves the blood and ly Occurs in 15-20% of AIDS patients. Caused by a herpes virus infection. Pulmonary involvement in 20-50%. Usually preceded by cutaneous involvement. Airway lesions visible at bronchoscopy. Coarse reticular opacities at the lung bases. Ill-defined or spiculated (flame-shaped) nodules. Parahilar or peribronchovascular distribution. Interlobular sepatl thickening. Pleural effusion. Lymph node enlargement.
127
Q

149.Regarding BAC: t/f

  1. The mucinous form rarely causes multifocal disease
  2. If solitary, is usually associated with mediastinal adenopathy
  3. Causes pesudocavitation
  4. Most common cause of pleural tags compared to other pulmonary cancers
  5. Most common presentation is solitary form
A

t 33.Causes pesudocavitation T - ie. dilatation of intact airspaces from desmosplastic reaction/bronchiectasis,/focal emphysema, occurs in 50-60%. True cavitation is unusual because of the relative lack of necrosis and the preservation os normal lung architecture and perfusion to the involved lung. Dahnert gives true cavitation as 7%, and BAC is the 2nd most common type associated with cavitation after SCC

  1. Regarding BAC: (GC)
  2. The mucinous form rarely causes multifocal disease F - more likely to be multicentric and diffuse. Poor prognosis (26% 5YS).
  3. If solitary, is usually associated with mediastinal adenopathy F - a solitary pulmonary nodule is more likely to be the non mucinous subtype, which is more often localised. Excellent prognosis (72% 5YS).
  4. Causes pesudocavitation T - ie. dilatation of intact airspaces from desmosplastic reaction/bronchiectasis,/focal emphysema, occurs in 50-60%. True cavitation is unusual because of the relative lack of necrosis and the preservation os normal lung architecture and perfusion to the involved lung. Dahnert gives true cavitation as 7%, and BAC is the 2nd most common type associated with cavitation after SCC.
  5. Most common cause of pleural tags compared to other pulmonary cancers T - “Rabbit ears” or pleural tags (“tail sign”) are seen in 55% of BAC; represent linear strands extending from nodule to pleura, histopathologically may be due to infiltative tumour growth, lyphamgitic spread, or desmoplastic reaction. Differential includes other peripheral tumours (adenocarcinoma, adenosquamous ca, SCC), infectious nodules (atypical MTB), and organising pneumonia/fibrosis where pleural tags represent areas of peribronchiolar inflammation and linear atelectasis.
128
Q

150.Regarding pleural effusion on antenatal scan: t/f

  1. Most commonly chylothorax
  2. Not necessarily indicative of poor outcome
  3. Associated with most cases of CCAM
A

t 1,2

  1. Regarding pleural effusion on antenatal scan: (GC)
  2. Most commonly chylothorax T - most common cause of isolated pleural effusion in the newborn.
  3. Not necessarily indicative of poor outcome T - 10% resolve spontaneously. May require thoracentesis or thoracoamniotic shunting if large and recurrent.
  4. Associated with most cases of CCAM F - 10% CCAM assoc with hydrops - where mechanism could produce pleural effusion. Remainder generally are isolated finding. (Blackbook)

CAUSE OF FETAL PLEURAL EFFUSIONS:
Unilateral usually due to lung masses: CHD, sequestration, CCAM. Bilateral: fetal hydrops, plumonary lymhangiectasia (rare) unilateral or bilateral: idiopathic, infection, chromosomal anomalies (21, Turner’s). [Primer]

129
Q

151.Regarding intralobar sequestration: t/f

  1. Usually has a pleural covering
  2. Is supplied by the bronchial arteries
  3. Commonly drains via the azygous vein
  4. Usually presents in infancy
  5. Sequestration demonstrates air bronchograms
A

everything is false

  1. Regarding intralobar sequestration (T/F): (TW)
  2. Usually has a pleural covering - F - enclosed by visceral pleura of affected pulmonary lobe but separated from bronchial tree. Lacks it’s own visceral pleura.
  3. Is supplied by the bronchial arteries F – supply distal thoracic aorta 73%; prox. abdominal aorta 22%.
  4. Commonly drains via the azygous vein - F - venous drainage is usually normal to the LA, although abnormal connections to the vena cava, azygous vein, or RA may occur.
  5. Usually presents in infancy - F - ELS more commonly Dx in newborns or infants. ILS usually presents in late childhood or adolescence with recurrent pulmonary infections (UTD). Adulthood 50%>20y (D).
  6. Sequestration demonstrates air bronchograms F – sequestration lacks normal communication with tracheobronchial tree (ILS and ELS), although ILS, albeit uncommon, may have abnormal connection. Note that both could potentially have air bronchograms as both can have connection to GIT
130
Q
  1. Regarding cavitating lung lesions, which is least likely:
  2. Laryngeal papilloma
  3. Lymphogranuloma venereum
  4. Hamartoma
A
  1. Lymphogranuloma venereum - F - STI caused by infection of lymphatic tissue by C.trachomatis; site of primary infection is usually around the genitals, sometimes the rectum (usu from anal intercourse). Present with extremely painful inguinal LNs and deep rectal ulceration; may progress to fistula, perirectal abscess, strictures. [RG 1996]
  2. Regarding cavitating lung lesions, which is least likely: (GC)
  3. Laryngeal papilloma - T - bimodal distribution: <10yo. have diffuse involvement usually from HPV-infected mother during birth; 21-50yo. usually have a single papilloma. Manifests as tracheobronchial or pulmonary papillomatosis; latter from aerial dissemination (bronchoscopy, tracheal intubation) ~10yrs after initial dx - appear as non-calcified granulomata progressing to cavitation. [Dahnert]
  4. Lymphogranuloma venereum - F - STI caused by infection of lymphatic tissue by C.trachomatis; site of primary infection is usually around the genitals, sometimes the rectum (usu from anal intercourse). Present with extremely painful inguinal LNs and deep rectal ulceration; may progress to fistula, perirectal abscess, strictures. [RG 1996]
  5. Hamartoma - F - most common benign tumour of lung. Fat density in 50% (diagnostic), calcification in 15-20% (popcorn is almost pathognomonic). Cavitation is extremely rare. Possibly option 2 may have been lymphomatoid granulomatosis (a polymorphic form of Angiocentric lymphoma). Histo: nodular and diffuse interstitial infiltration by lymphoid cells; tends to have an angiocentric and angiodestructive pattern. As the nodules enlarge, central necrosis and cavitation develop. A large vessel may be present in the center of the nodule as well as infiltration by lymphoid cells, justifying the descriptive designation of angiocentric lymphoma. IHC studies show that most of the lymphoid cells are CD4-positive T cells. Imaging: non-specific bilateral soft tissue nodules, predominantly basal, cavitary in 25%. [Unusual primary lung tumours RG 2002]
131
Q

153.AML (Best prognostic factor)

  1. Philadelphia chromosome
  2. most common in children
A

Probably AML in children

Philadelphia chromosome- present in CML > AML > ALL- translocation between 9 and 22- forms the BCR ABL 1 -> causes tyrosine kinase activation-> causes continuous cellular proliferation- can be target by tyrosine kinase inhibitorFor CML- Ph + tumour -> more likely to have blastic transformation BUT have longer lifespan due to tyrosine kinase inhibitorFor AML- Ph + = more likely to have induction failure and relapse

132
Q

160.Asbestos __ores most likely found in :

  1. malignant mesothelioma
  2. pleural fluid
  3. pleural plaque
  4. parenchymal biopsy
  5. bronchial washings
A
  1. parenchymal biopsy - T - diffuse pulmonary interstitial fibrosis (asbestosis) - and changes are indistinguishable from those resulting form other causes of diffuse interstitial fibrosis except for the presence of asbestos bodies.
  2. Asbestos __ores most likely found in :
  3. malignant mesothelioma
  4. pleural fluid - F - Uncommonly, asbestos exposure induces pleural effusions, which are usually serous, but may be bloody.
  5. pleural plaque - F - most common manifestatio of asbestos exposure, however do not contain asbestos bodies.
  6. parenchymal biopsy - T - diffuse pulmonary interstitial fibrosis (asbestosis) - and changes are indistinguishable from those resulting form other causes of diffuse interstitial fibrosis except for the presence of asbestos bodies.
  7. bronchial washings
133
Q

104.Definition of Bulla

A

1.>1cm and <1mm thick wall

134
Q
  1. Bordetella pertussis t/f
  2. exotoxin
  3. cause laryngotracheobronchitis
  4. peripheral lymphocytosis
  5. membrane
A

Bordetella pertussis

  1. exotoxin - T - Multiple toxins.
  2. cause laryngotracheobronchitis - *AJL - F - AKA croup, this is caused by RSV or parainfluenza virus. [acute laryngotracheobronchitis]
  3. peripheral lymphocytosis - T - predominant nonspecific laboratory finding of B. pertussis is an overall leukocytosis due to lymphocytosis.
  4. membrane - T - B. pertussis are small, gram-negative (ie cell membrane + cell wall), pleomorphic coccobacilli.
135
Q

165.Tracheo-oesophageal fistula - list the type and frequency

A
Atresia with distal fistula 85 %
Only atresia 8 %
Only fistula (H type) 4 % 
Atresia with proximal and distal fistula 1%
Atresia with proximal fistula 1%
136
Q
  1. Cystic fibrosis.
  2. Defective membrane transport of sodium
  3. Defective membrane transport of chloride
A

2 true
CF- CFTR gene mutation- causes abnormality of CFTR transporter- skin: no CL ( and thus Na) reabsorption -> salty skin - > salt test- in other mucosa : too much Cl reabsorption (and thus Na -> very non-salty secretion -> easily dries up and becomes very mucous

137
Q
  1. Legionella is a :
  2. gram negative bacilli
  3. Sporing coccus
  4. Protozoal
  5. Fungus
  6. Helminth
  7. Spore forming bacteria
A

1.gram negative bacilli - T - Legionellaceae family consists of 50 species and includes more than 70 serogroups. L. pneumophilia is the most common species, which causes at least 80% of human infections. Legionella species are aerobic, gram-negative bacilli.

138
Q
  1. Concerning centrilobular emphysema, which of the following is most correct:
  2. The acini are uniformly enlarged from level of terminal bronchiole to terminal alveoli
  3. Proximal part of acini is enlarged, relative or complete sparing of distal acini
  4. Proximal portion of acinus normal or near normal, with dominant involvement of distal portion
  5. Whole acini destroyed leaving irregular lined spaces >1cm in diameter
  6. Acini uniformly involved but disease effects central zone of secondary pulmonary lobules
A

b Proximal part of acini is enlarged , relative or complete sparing of distal acini. T

  1. Concerning centrilobular emphysema, which of the following is most correct: (JS)

a The acini are uniformly enlarged from the level of terminal bronchioles to terminal alveoli. F - Panacinar/panlobular

b Proximal part of acini is enlarged , relative or complete sparing of distal acini. T

c Proximal part of the acinus normal or near normal with dominant involvement of distal portion. F - Paraseptal/distal acinar

d Whole acini destroyed leaving irregular spaces > 1cm in diameter. F - can occur with both panacinar and centrilobular - if severe

e Acini uniformly involved but disease effects central zone of secondary pulmonary lobule. F

139
Q
  1. Pulmonary adenocarcinoma vs squamous cell carcinoma. Which of the following is most correct

:1. The 2 are macroscopically similar

  1. Adenocarcinomas are more likely to show cavitation
  2. Adenocarcinoma is more likely to be peripheral and affect women, rather than SCC
  3. Adenocarcinoma have a stronger association with emphysema, chronic bronchitis
  4. Adenocarcinoma is more likely to show massive lymphadenopathy and extrathoracic disease compared with SCC
A
  1. Adenocarcinoma is more likely to be peripheral and affect women, rather than SCC T - 75% are peripheral, most common tumours to affect women and non smokers (but also seen in smokers). SCCs tend to be central, more common in male smokers.
  2. Pulmonary adenocarcinoma vs squamous cell carcinoma. Which of the following is most correct: (GC)
  3. The 2 are macroscopically similar F - adenocarcinoma usually peripheral, may be associated with scarring, 80% contain mucin. SCC appears as grey-white infiltrative tumour, usually associated with larger, more central bronchi, cavitation in 30%.
  4. Adenocarcinomas are more likely to show cavitation F - cavitating tumours = 4/5 SCC, followed by BAC. [Dahnert]
  5. Adenocarcinoma is more likely to be peripheral and affect women, rather than SCC T - 75% are peripheral, most common tumours to affect women and non smokers (but also seen in smokers). SCCs tend to be central, more common in male smokers.
  6. Adenocarcinoma have a stronger association with emphysema, chronic bronchitis F - implies a heavy smoking history, thus small cell and SCC would have a stronger association.
  7. Adenocarcinoma is more likely to show massive lymphadenopathy and extrathoracic disease compared with SCC ?? T - SCC typically a large central mass that spreads to local hilar nodes, but disseminates outside thorax later than other types. Adeno’s metastasise widely at an early stage.
140
Q
  1. Patient with markedly thickened pleura encasing one lung. Which of the following statements is most correct?
  2. Only approximately 10-20% with patients with mesothelioma will have associated lower lobe fibrosis
  3. Mesothelioma and metastatic adenocarcinoma have markedly different appearances on light microscopy
  4. Presence of asbestos bodies on pleural biopsy suggest change is more likely reactive / fibrosis
  5. Granuloma in pleural tissue suggests chronic irritation rather than neoplastic infective etiology
  6. Apical lung lesion suggests it is most likely infective in nature
A
  1. Only approximately 10-20% with patients with mesothelioma will have associated lower lobe fibrosis T - 90% of malig. mesotheliomas are related to asbestos but only 10% have asbestosis. As opposed to peritoneal mesothelioma, where 50% have asbestosis. [Robbins]
  2. Patient with markedly thickened pleura encasing one lung. Which of the following statements is most correct? (GC)
  3. Only approximately 10-20% with patients with mesothelioma will have associated lower lobe fibrosis T - 90% of malig. mesotheliomas are related to asbestos but only 10% have asbestosis. As opposed to peritoneal mesothelioma, where 50% have asbestosis. [Robbins]
  4. Mesothelioma and metastatic adenocarcinoma have markedly different appearances on light microscopy F - differentiated by antigenic keratin positivity, CEA stain negative, presence of long microvilli on EM.
  5. Presence of asbestos bodies on pleural biopsy suggest change is more likely reactive / fibrosis F - diffuse pleural thickening is rare, involves visceral pleura (cf. pleural plaques - parietal). Neither of theses entities (or mesothelioma) contain asbestos bodies on histology.
  6. Granuloma in pleural tissue suggests chronic irritation rather than neoplastic infective etiology F
  7. Apical lung lesion suggests it is most likely infective in nature F
141
Q
  1. Ignoring thoracic nodes, re bronchogenic cancer met order
  2. Bone, liver, lung
  3. Adrenal, liver brain, bone
  4. Lungs, liver, adrenal, bone
  5. Brain, bone, liver, adrenal
  6. Or some other combination
A
  1. Adrenal, liver brain, bone - T - direct from path notes: common sites: adrenal >50%, liver 30-50%, brain 20%, bone 20%.
142
Q
  1. sarcoid – least likely to affect
  2. cerebrum
  3. gallbladder
  4. lungs
A
  1. gallbladder - rare. Of the GIT stomach is the most common to be affected, and of interest 0.5% affect scrotum.
  2. sarcoid – least likely to affect (TW)
  3. cerebrum - CNS 9%
  4. gallbladder - rare. Of the GIT stomach is the most common to be affected, and of interest 0.5% affect scrotum.
  5. lungs - thoracic sarcoid 90% of which parenchymal disease 60%.
143
Q
  1. Cryptococcus in immunocompromised
  2. Basal exudate
  3. Single coin lesion
  4. Military lung
  5. End arteritis
A
  1. Military lung - T - In patients with AIDS, a diffuse interstitial pattern, variously described as reticular or nodular, resembling P. jiroveci pneumonia, may be seen. A miliary pattern may occur, as may single or multiple nodules, occasionally with cavitation. Lymph node enlargement is uncommon, as is pleural effusion. In healthy patients, cryptococcal infection usually manifests as one or more peripheral, circumscribed nodules, usually without cavitation. Less commonly, air-space consolidation is seen.
144
Q
  1. Mucor – not typical
  2. Diabetics and immunosuppressed
  3. Lots of small nodules in brain
  4. Sinus disease
  5. Bowel wall thickening
A
  1. Lots of small nodules in brain - ?F - CNS infection typically arises from an adjacent paranasal sinus infection (typically rhinocerebral) causing meningoencephalitis sometimes complicated by cerebral infarctions when fungi invade arteries and induce thrombosis. (Robbins)
  2. Mucor – not typical (TW)
  3. Diabetics and immunosuppressed - T - immunocompromised patients / diabetics.
  4. Lots of small nodules in brain - ?F - CNS infection typically arises from an adjacent paranasal sinus infection (typically rhinocerebral). However, there have been cases of isolated CNS zygomycosis described (~30), infection thought to result from seeding of the brain during episode of fungemia (?nodules).
  5. Sinus disease - T - most common clinical presentation of zygomycosis is rhino-orbital-cerebral infction. Usually presents as acute sinusitis with fever, nasal congestion, purulent nasal discharge, headache, and sinus pain.
  6. Bowel wall thickening - T - Intestinal zygomycosis is manifested by erosive necrotic ulcers with thromboses and gangrene in the GIT from the oesophagus to colon. These lesions may perforate GI wall leading to peritonitis. Death usually caused by shock from hemorrhage of the bowel resulting in peritonitis and bowel infarction.

Zygomycetes (Mucomycosis) = class of fungi that can cause a variety of infections if humans, particularly in immunocompromised patients and those with diabetes. Zygomycetes are ubiquitous in nature, and can be found on decaying vetetation and in the soil. Grow rapidly and release large numbers of spores that can become airborne. All humans have ample exposure to these fungi during day-to-day activities. The fact that zygomycosis is a rare human infection reflects the effectiveness of the intact human immune system. Genera most commonly found in human infections - Rhizopus, Mucor, and Cunninghamella. (UTD)

145
Q
  1. Which is not true regarding Infections?

a Klebsiella, Staphylococcal aureus and pneumococcus can form lung abscesses.
b Mycoplasma typically occurs in elderly.
c Haemophilus type b occurs in neonates.
d Aspiration gives sterile fluid.

A

Mycoplasma typically occurs in elderly. F - most common in 5-20yo.

  1. Which is not true regarding Infections? (GC)

a Klebsiella, Staphylococcal aureus and pneumococcus can form lung abscesses. T - pneumococcus can rarely cause abscess (type 3). More commonly due to (or with additional) anaerobic infection.

b Mycoplasma typically occurs in elderly. F - most common in 5-20yo.

c Haemophilus type b occurs in neonates. ?? - I assume this applies to pulmonary infections. Hib is a cause of neonatal bacteraemia and meningitis; and it did cause up to one-third of the documented cases of childhood pneumonia in the prevaccine era [eMedicine]. Group B strep is the most common cause of neonatal pneumonia (50% mortality in LBW infants); others include pneumococcus, listeria, candida, chlamydia trachomatis.

d Aspiration gives sterile fluid. ?? check this ?? - a parapneumonic effusion is a transudate, and therefore would be aseptic. On the other hand, an empyema is an exudate due to microbial invasion through either direct extension of a pulmonary infection or blood-borne seeding. Culture of infected pleural fluid yields positive results in ~60% of cases [eMedicine: pleural effusions].

146
Q
  1. Features of lung carcinoma, which is true:

a Extrathoracic spread by squamous cell ca before presentation.
b alaveolar cell Ca <2% are focal.
c Adenocarcinoma can have dilated bronchi and consolidation.
d Stage 3 disease surgery contraindicated.

A

**LW:
Favored option is C:
c Adenocarcinoma can have dilated bronchi and consolidation.
invasive mucinous adenocarcinoma have air bronchograms and consolidation.

While extra thoracic spread is seen more commonly with adenocarcinoma compared to SCC, while SCC will more commonly have hilar and mediastinal involvement at presentation.

Previous answer:
a Extrathoracic spread by squamous cell ca before presentation. T - 60-80% have early mets at time of diagnosis; should be regarded as systemic disease regardless of stage.

  1. Features of lung carcinoma, which is true: (GC)

a Extrathoracic spread by squamous cell ca before presentation. T - 60-80% have early mets at time of diagnosis; should be regarded as systemic disease regardless of stage.
*LW: cant find a reference from this…thought (possibily incorrectly) that adeno tended to present more commonly with extra thoracic disease than SCC.

b Alveolar cell Ca <2% are focal. F - 60-90% are localised; with 43% occurring as a single mass, remainder as ground glass attenuation. Diffuse form (either diffuse consolidation, lobar form, or multiple nodules) in 10-40%.

c Adenocarcinoma can have dilated bronchi and consolidation. F
*LW –> cant invasive mucinous adenocarcinoma have air bronchograms and consolidation???

d Stage 3 disease surgery contraindicated. F -

resectable if stage IIIA: T1-3, N1-2, M0.
Unresectable if stage IIIB (ie. T4 or N3) or stage 4 (M1). [Dahnert]

147
Q
  1. Regarding eosinophilic granuloma, which is wrong?
  2. Bone scan negative in 1/3
  3. Birbeck granules would be a feature
  4. Reactive non-neoplastic proliferative disorder
  5. Most aggressive form of LCH
  6. Lungs involved 20%
  7. Vertebra plana is a feature
A

Most aggressive form of LCH - F - ?

  1. Regarding eosinophilic granuloma, which is wrong? (TW)
  2. Bone scan negative in 1/3 - T - negative bone scans in 35%. Radiographs more sensitive.
  3. Birbeck granules would be a feature - T - Classic EM finding of pentalaminar Birbeck granules (in langerhan cells). These are infoldings of the cell membrane, possibly because of antigen processing. Granule protein has been isolated, and a gene “Langerin” has been cloned (UTD).
  4. Reactive non-neoplastic proliferative disorder - T - Althougth LCH is characterised by a clonal proliferation of cells, it is likely that LCH in adults represents an abnormal immune response in response to an unidentified antigenic stimulus rather than a neoplasm (TI). Most investigators consider LCH to result from immunologic dysfunction (UTD).
  5. Most aggressive form of LCH - F - ?
  6. Lungs involved 20% - T - Lung involvement in multisystem LCH is 20-40%.
  7. Vertebra plana is a feature - T Historical terms histiocytosis-X, eosinophilic granulmoa, Letterer-Siwe disease, Hand-Schuller-Christian disease, and diffuse reticuloendotheliosis should be abandond in favor of the term LCH.